You are on page 1of 67

VISION IAS ™ ...

Inspiring Innovation
www.visionias.in
www.visionias.wordpress.com

“The significant problems we face cannot be solved at the same level of thinking we were at when we created them." - Albert Einstein
ANALYSIS / APPROACH / SOURCE / STRATEGY: GENERAL STUDIES PRE 2017 PAPER
- TEAM VISION IAS

 The analysis is based on premise that UPSC


wants to identify aspirants with certain skills
and aptitude required to function as a good
administrator. UPSC does not explicitly
describe these skills so we infer them from
Syllabus, Question Paper and feedback of
experts and students.
 This analysis is an attempt to quantify the
subjective understanding and make explicit
the implicit.
 We have inferred some conclusions from these
stats but you are free to infer differently.
Moreover independent/impartial thinking is
one of the qualities expected in
administrators.

1 www.visionias.in ©Vision IAS


Observations on CSP 2017

 This year’s paper was more conceptual compared to the papers of CSAT era and
it displayed a good mix of current affairs and conventional subjects.
 The level of questions ranged from easy to moderate and difficult from almost
in all the sections .The elimination was difficult as the options framed were too
close or very similar.
 However, even in conventional or static portion the tilt was more towards
Polity and Governance. Polity questions were asked from traditional concepts
as compared to previous year’s trend. Eg. concept of rights, democracy, etc.
 In the current affairs section, the testing was done from their basic concepts as
well as facts related to them. However, the questions from this section was not
restricted to the present year but from the previous 2 – 3 years.
 Economy questions were largely drawn from Current Affairs eg- GST, Monetary
Policy Committee, etc. but conventional understanding in questions like post
1991 reforms, etc was also checked.
 Geography and Environment sections were difficult to attempt and required
conceptual clarity, linkage of current affairs with basic concepts (e.g. Indian
Ocean Dipole), detailed reading of Atlas, Conventions/Alliance to which India is
not party were also asked (e.g GCCA, etc), the questions on unique features of
geographical locations were also focused (eg Chandipur, Sathyamangalam Tiger
Reserve).
 The overall weightage to History sections has remained more or less same but
difficulty level has increased in terms of factual knowledge and standard
sources.
 S& T questions were largely drawn from news items.
 Govt initiatives ranging from education, health and nutrition, etc. were again
focused this year but solving them required deeper knowledge.
 Deceptive presentation is a common method to segregate deep knowledge
with superficial knowledge or hunch. This technique was used in questions like
National Investment and Infrastructure Fund (4,00,000 crores), National
Pension Scheme( age group), etc.

2 www.visionias.in ©Vision IAS


 Pointers for most of the questions could be traced to standard sources such as
The Hindu, Indian Express, PIB, etc. But as recommended earlier, in the age of
internet, complete knowledge of topic is expected.

Recommendations:

 In a difficult paper, the static-conventional part is the savior. Thoroughly read


basic/standard books to be very clear of basic concepts. One good News Paper
(like The Hindu, Indian Express, etc) and sites like PIB, PRS must be followed.
One shall also read India Year Book, as many of the questions related to
government initiatives could be answered based on that.
 Regularly follow the current issues and news. One should combine reading
newspaper with simultaneous revision of related conceptual key terms.
 Read one good book on each traditional subject. This will help in prelims,
mains and interview i.e. all stages of examination. This will also help in making
some unapproachable question approachable.
 Be ready for both analytical and factual questions at the same time. This year
there is a fair combination of both.
 In this year’s paper, where difficulty level is on the higher side, number of
attempted Q's also matter. In case of confusing options, one cannot go on
attempting all as has been the case in previous years. Negative marking takes
a heavy toll.
 Strategy should be to avoid negative marking in tough questions and maximize
your score by first finding the easy ones and completing them.
 Read questions carefully and use elimination technique wherever possible.
 One should always have few strong areas to rely on. Do not ignore your strong
areas to do research in weak areas.
 Solving previous year’s questions would help in identifying the key areas
which UPSC focuses on. Many of the topics from static portion (like Trade
disputes act, liberalization etc.) have been asked earlier by UPSC.

3 www.visionias.in ©Vision IAS


EM = Essential material like basic books etc.
RM = Reference material
EN = Essential News/Current Affairs
RR = Random Read like random website etc.
E : Easy , M : Medium , D : Difficult

Nature of Question
F: Fundamental, Conventional and conceptual question which is easily available
in commonly recommended books. If a current affairs source is mentioned then
it indicates that you had one more reason to prepare this.
FA: Fundamental Applied question is an analytical question which requires
information + application of mind.
Answer to these question are from commonly recommended books but not so
obvious to find.
CA: Current Affair question which can be answered almost completely using
given source alone.
CAA: Current Affair Applied question needs information from more than one
source to answer AND/OR application of mind (common sense and overall
understanding) of aspirant. This is why source for these questions may not be
answering the question completely.
FCA: Fundamental + Current affair. This is a F or FA question supplemented with
current affairs or a current affairs question that needs background information
for complete answer. Source to these questions might not answer the questions
in entirety.
U (Unconventional Question): Unconventional Question means the question
that is distantly related to syllabus provided by UPSC. It is neither present in even
reference sources (in addition to recommended books) nor prominently in news.

4 www.visionias.in ©Vision IAS


A
L
n
e
s Nat Source
QN Section Question Explanation v Source Motivation Vision IAS
w ure Type
e
e
l
r
1 Medieval Which one of the following B Motupalli was the chief port of D U Early India: RM To check in-
History was a very important seaport the Kakatiyas and this port was From the depth
in the Kakatiya kingdom? visited by the Venitian traveller, Origins to A.D. knowledge on
(a) Kakinada Marco Polo. 1300, Part 1 Medieval
(b) Motupalli Romilla Indian history
(c) Machilipatnam Thapar page
(Masulipatnam) 384
(d) Nelluru
2 Environment With reference to ‘Global A The Global Climate Change D CA http://www.g RR To check
Climate Change Alliance’, Alliance (GCCA) was established cca.eu/about- about the
which of the following by the European Union (EU) in the- Institutions
statements is/are correct? 1. 2007 to strengthen dialogue and gcca/what-is- related to
It is an initiative of the cooperation with developing the-gcca climate
European Union. countries, in particular least change and
2. It provides technical and developed countries (LDCs) and climate
financial support to targeted small island developing States change
developing countries to (SIDS). Under the first pillar, the financing
integrate climate change into GCCA+ serves as a platform for mechanism
their development policies dialogue and exchange of
and budgets. experience between the EU and
3. It is coordinated by World developing countries, focusing on
Resources Institute (WRI) climate policy and bringing
and World Business Council renewed attention to the issue of
for Sustainable Development international climate finance.
(WBCSD). Under the second pillar, the
Select the correct answer GCCA+ acts as a source of
using the code given below: technical and financial support for
(a) 1 and 2 only the world's most climate-
(b) 3 only vulnerable countries, whose
(c) 2 and 3 only populations need climate finance
(d) 1, 2 and 3 the most.
International organisations –
notably the Centre for
International Forestry Research
5 www.visionias.in ©Vision IAS
(CIFOR), the Food and Agriculture
Organisation (FAO), the United
Nations Capital Development
Fund (UNCDF), the United Nations
Development Programme (UNDP),
the United Nations Environment
Programme (UNEP), UN Habitat
and the World Bank – are involved
in the implementation of some
GCCA-funded interventions, and
also co-finance some initiatives.
Statement 3 is not correct:
Through the Greenhouse Gas
Protocol (GHGP) World Resources
Institute (WRI) and the World
Business Council for Sustainable
Development (WBCSD) work with
businesses to develop standards
and tools that help companies
measure, manage, report and
reduce their carbon emissions.
3 Art and With reference to the B Statement 1 is not correct: D U THE WONDER RM To check in-
Culture religious history of India, Sautrantika and Sammitya were THAT WAS depth
consider the following the the sects of Buddhism. INDIA: A L knowledge of
statements: Statement 2 is correct: Basham page religious
1. Sautrantika and Sammitiya Sarvastivadins (They who say "All 272 philosophies.
were the sects of Jainism. is"), had the view that the
2. Sarvastivadin held that the constituents of phenomena
constituents of phenomena (dharmas) were not wholly
were not wholly momentary, momentary, but existed forever in
but existed forever in a a latent form.
latent form.
Which of the statements
given above is/are correct?
(a) 1 only
(b) 2 only
(c) Both 1 and 2
(d) Neither 1 nor 2
4 Geography Mediterranean Sea is a C The countries with coastlines on M FCA Atlas EM These Map on Page 35, 3.1 Israel-
border of which of the the Mediterranean Sea are countries are Palestine, PT-365 International
following countries? Albania, Algeria, Bosnia and often seen in Relations.
6 www.visionias.in ©Vision IAS
1. Jordan Herzegovina, Croatia, Cyprus, news as they
2. Iraq Egypt, France, Greece, Israel, Italy, are regions
3. Lebanon Lebanon, Libya, Malta, Morocco, under conflict
4. Syria Monaco, Montenegro, Slovenia, facing acute
Select the correct answer Spain, Syria, Tunisia and Turkey. In refugee crisis.
using the code given below: addition, the Gaza Strip and the
(a) 1, 2 and 3 British Overseas Territories of
(b) 2 and 3 only Gibraltar and Akrotiri and
(c) 3 and 4 only Dhekelia have coastlines on the
(d) 1, 3 and 4 only sea.
On the Eastern shore, Turkey,
Syria, Lebanon and Israel have
their coastline on the
Mediterranean Sea.
5 Current With reference to ‘National D National Investment and M CAA http://www.t EN Infrastructure
Affairs Investment and Infrastructure Fund (NIIF) is a fund hehindu.com/ has been a
Infrastructure Fund’, which created by the Government of business/Indu very
of the following statements India for enhancing infrastructure stry/Govt.- important
is/are correct? financing in the country. Its mulls- focus area of
1. It is an organ of NITI creation was announced in the %E2%80%98o the
Aayog. Union Budget 2015-16. Governing utcome- government
2. It has a corpus of Rs. Council of NIIF is an advisory body linked%E2%80 and therefore
4,00,000 crore at present. - chaired by the Finance Minister. %99- NIIF has
Select the correct answer It is not an organ of NITI Aayog. incentive-for- remained
using the code given below: Hence statement 1 is not correct. NIIF- continuously
(a) 1 only The proposed corpus of NIIF is Rs. chief/article14 in news.
(b) 2 only 40,000 Crores (around USD 6 378399.ece
(c) Both 1 and 2 Billion). Hence statement 2 is also http://arthape
(d) Neither 1 nor 2 not correct. dia.in/index.p
hp%3Ftitle%3
DNational_Inv
estment_and_
Infrastructure
_Fund_(NIIF)
http://niifindi
a.com/index.h
tml
6 Current The Global Infrastructure B The GIF supports Governments in D CAA http://www.t EN This is an old
Affairs Facility is a/an bringing well-structured and hehindu.com/ news,
(a) ASEAN initiative to bankable infrastructure projects business/Econ however is
upgrade infrastructure in to market. GIF’s project support omy/gif- important
7 www.visionias.in ©Vision IAS
Asia and financed by credit can cover the spectrum of design, launched-to- considering
from the Asian Development preparation, structuring and help-india- the focus on
Bank. transaction implementation bridge-infra- infrastructure
(b) World Bank collaboration activities, drawing on the deficit/article6 and its
that facilities the preparation combined expertise of the GIF’s 486440.ece financing in
and structuring of complex Technical and Advisory Partners India
infrastructure Public-Private and focusing on structures that
Partnership (PPPs) to enable are able to attract a wide range of
mobilization of private sector private investors.
and institutional investor Along with the World Bank Group,
capital. Canada is co-chair of the GIF’s
(c) Collaboration among the Governing Council.
major banks of the world
working with the OECD and
infrastructure projects that
have the potential to
mobilize private investment.
(d) UNCTAD funded initiative
that seeks to finance and
facilitate infrastructure
development in the World.
7 Polity and For election to the Lok C Qualifications for election to Lok E F Laxmikant - EM to check the All India test series - test 2115
Governance Sabha, a nomination paper Sabha: Indian Polity, basic A person "X" is a 30 year old
can be filed by - must be a citizen of India chapter - knowledge citizen of India and is a
(a) anyone residing in India. - not less than 25 years of age "Parliament" registered elector in
(b) a resident of the - must be registered as an elector Parliamentary constituency. "X"
constituency from which the for a parliamentary constituency is qualified for which of the
election is to be contested. following positions?
(c) any citizen of India whose 1. President
name appears in the 2. Prime Minister
electoral roll of a 3. Speaker of Lok Sabha
constituency. 4. Chairman of Rajya Sabha
(d) any citizen of India. Select the correct answer using
the code given below.
(a) 1 and 2 only
(b) 2 and 3 only
(c) 3 and 4 only
(d) 2, 3 and 4 only
8 Geography Consider the following B Statement 1 is not correct: In D FA Atlas EM In depth
statements: India, the Himalayas are spread knowledge
1. In India, the Himalayas are over Jammu & Kashmir, Himachal and detailed
8 www.visionias.in ©Vision IAS
spread over five states only. Pradesh, Uttarakhand, Sikkim, observation of
2. Western Ghats are spread Assam, West Bengal, Arunachal Atlas is tested.
over five states only. Pradesh. Some extensions of
3. Pulicat Lake is spread over Shiwaliks are also present in
two States only. Punjab and Haryana.
Which of the statements Statement 2 is not correct:
given above is/are correct? Western Ghats are spread over six
(a) 1 and 2 only states, namely- Gujarat,
(b) 3 only Maharashtra, Goa, Karnataka,
(c) 2 and 3 only Kerala, Tamil Nadu.
(d) 1 and 3 only Statement 3 is correct: Pulicat,
the second largest brackish water
lake in India, lies on the border of
Andhra Pradesh and Tamil Nadu.
Over 40% of it lies in Andhra
Pradesh and 60% in Tamil Nadu.
9 Environment Biological Oxygen Demand C Biological oxygen demand (BOD) E F NCERT 12 - EM To check the Asked in All India Test Series
(BOD) is a standard criterion is the amount of dissolved oxygen Biology, basic concepts tests - 2113, 2126, 2358 Q.
for needed (i. e., demanded) by chapter 16 of ecology and Consider the following
(a) Measuring oxygen levels aerobic micro organisms to break environment. statements with regard to Bio-
in blood down organic material present in chemical Oxygen Demand
(b) Computing oxygen levels a given water sample. A highly (BOD):
in forest ecosystems polluted water will have a high 1. It is the amount of oxygen
(c) Pollution assay in aquatic levels of biological oxygen needed by aerobic micro
ecosystems demand (BOD). organisms to break down
(d) Assessing oxygen levels in organic material present in a
high altitude regions given water sample.
2. Higher level of BOD indicates
lower pollution level of the
water.
Which of the statements given
above is/are correct?
(a) 1 only
(b) 2 only
(c) Both 1 and 2
(d) Neither 1 nor 2
10 Current With reference to the role of B Statement 1 is correct: UN- M CAA http://www.t EN It was in news Covered in PT 365 Updation –
Affairs UN-Habitat in the United Habitat is the United Nations hehindu.com/ due to two Environment
Nations Programme working programme working towards a news/national factors:
towards a better urban better urban future. Its mission is /india-elected- 1. In October
future, which of the to promote socially and president-of- 2016, at the
9 www.visionias.in ©Vision IAS
statements is/are correct? environmentally sustainable un- UN
1. UN-Habitat has been human settlements development habitat/article Conference on
mandated by the United and the achievement of adequate 18413884.ece Housing and
Nations General Assembly to shelter for all. Statement 1 is http://www.t Sustainable
promote socially and correct. hehindu.com/ Urban
environmentally sustainable Statement 2 is not correct: It opinion/editor Development
towns and cities to provide partners with a range of ial/Making- – Habitat III –
adequate shelter for all. organizations in its endevaour. For cities- member
2. Its partners are either example, governments, local inclusive/articl states signed
governments or local urban authorities, NGOs, Trade unions, e16079545.ec the New
authorities only. professionals, Academics and e Urban
3. UN-Habitat contributed to Researchers, Human Solidarity https://unhabi Agenda. This
the overall objective of the Groups, Indigenous People, tat.org/about- conference
United Nations system to Private Sector, Foundations, us/un-habitat- happens once
reduce poverty and to Financial Institutions. at-a-glance/ in 20 years.
promote access to safe Statement 3 is correct: In the 2. In May
drinking water and basic absence of effective urban 2017, India
sanitation. planning, the consequences of was elected as
Select the correct answer this rapid urbanization can lead to the president
using the code given below: lack of proper housing and growth of the UN-
(a) 1, 2 and 3 of slums, inadequate and out- Habitat. On
(b) 1 and 3 only dated infrastructure – be it roads, behalf of
(c) 2 and 3 only public transport, water, India, Housing
(d) 1 only sanitation, or electricity – and Urban
escalating poverty and Poverty
unemployment, safety and crime Alleviation
problems, pollution and health Minister will
issues. UN-Habitat contributes to preside over
the overall objective of a the meetings
sustainable urbanization. of UN-Habitat.
11 Current With reference to ‘National B The National Skills Qualifications M CAA http://www.t EN Many Covered in PT 365 Economy
Affairs Skills Qualification Framework (NSQF) organizes hehindu.com/ programmes
Framework (NSQF)’, which of qualifications according to a series features/educ have been
the statements given below of levels of knowledge, skills and ation/Bridge- launched by
is/are correct? aptitude. These levels are defined the-skills- the
1. Under NSQF, a learner can in terms of learning outcomes gap/article145 government
acquire the certification for which the learner must possess 56912.ece for skill
competency only through regardless of whether they were http://pib.nic.i development
formal learning. acquired through formal, non- n/newsite/pri in the country.
2. An outcome expected formal or informal learning. ntrelease.aspx In this regard,
from the implementation of It provides for multiple pathways, ?relid=157637 NQSF has

10 www.visionias.in ©Vision IAS


NSQF is the mobility horizontal as well as vertical, both http://www.n remained
between vocational and within vocational education and sda.gov.in/nsq regularly in
general education. vocational training and among f.html the news.
Select the correct answer vocational education, vocational
using the code give below: training, general education and
(a) 1 only technical education, thus linking
(b) 2 only one level of learning to another
(c) Both 1 and 2 higher level. Thus Statement 1 is
(d) Neither 1 nor 2 not correct and statement 2 is
correct.
12 Modern In the context of Indian D The Government of India Act of M F NCERT- RR With
History history, the principle of 1919 was enacted to satisfy the Textbook- reference to
‘Dyarchy (diarchy)’ refers to people of India to some extent. Chapter-1-to- Indian history
(a) Division of the central The salient features of the Act 8-Indian- Government
legislature into two houses. were as follows : (a) Preamble: Constituition- of India act
(b) Introductions of double The Act provided for a Preamble and- 1919 was one
government i.e., Central and that laid down the basic principles Administratio of the
Statement governments. and policies upon which it was n-Class-11 important act.
(c) Having two sets of rulers; based. According to it the policy Thus the sole
one in London and another of the British Parliament was – (i) criteria is to
in Delhi. to provide for the increasing check the
(d) Division of the subjects association of Indians in every basic
delegated to the provinces branch of Indian administration, knowledge of
into two categories. (ii) to develop self governing subject and
institutions with a view to the topic.
progressive realisation of
responsible government in British
India as an integral part of the
empire; (iii) accordingly, the
Preamble suggested for a
decentralised unitary form of
government. The Act divided the
functions of government in two
categories: central and provincial.
The provincial subjects were
further subdivided into
transferred and reserved. Thus,
in the provinces a new form of
government, dyarchy, was
introduced. Dyarchy means dual
set of governments, e.g.

11 www.visionias.in ©Vision IAS


accountable and non-
accountable.
In the transferred subjects the
Governors were to be assisted by
the ministers responsible to the
legislature while in the reserved
subjects the Governors were to be
advised by the councillors who
were not accountable to the
legislature.
13 Current Consider the following in B National Career Service (NCS) M CAA http://pib.nic.i EN Lack of Covered in PT 365 Economy
Affairs respect of ‘National Career project is an initiative launched by n/newsite/Pri employment
Services’: the Ministry of Labour and ntRelease.asp has remained
1. National Career Service is Employment (India) Government x?relid=15449 continuously
an initiative of the of India as a Mission Mode Project 8 in the news.
Department of Personnel for establishing quick and efficient https://www. Though, NCS
and Training, Government of career related services. Hence ncs.gov.in/Pag was
India. statement 1 is not correct. es/default.asp established in
2. National Career Service The main stakeholders for the NCS x 2015, it has
has been launched in a would include: http://dget.ni remianed in
Mission Mode to improve - Unemployed candidates seeking c.in/content/s news due to
the employment jobs tudents/natio focus on
opportunities to uneducated - Students seeking career nal-career- helping
youth of the country. counseling service.php people in
Which of the above - Candidates seeking vocational / finding
statements is/are correct? occupational guidance employment.
(a) 1 only - Illiterate, under-privileged
(b) 2 only sections of society, blue-collar
(c) Both 1 and 2 workers seeking placements and
(d) Neither 1 nor 2 guidance
- Person with different abilities
(PWDs), ex-servicemen, veterans /
senior citizens, etc.
- Employers seeking suitable
candidates
Thus statement 2 is correct.
14 Current Which of the following B The S4A by RBI envisages E CA http://www.t EN Stressed Covered in PT 365 Economy
Affairs statements best describes determination of the sustainable hehindu.com/ assets and Section and All India Test
the term ‘Scheme for debt level for a stressed borrower, business/Indu NPAs have series - 2128, 2222, 2241
Sustainable Structuring of and bifurcation of the outstanding stry/First-S4A- constantly Which of the following is/are
Stressed Assets (S4A)’, debt into sustainable debt and debt-recast- remained in among the steps taken by RBI
12 www.visionias.in ©Vision IAS
recently seen in the news? equity/quasi-equity instruments gets- the news. RBI to improve the stressed assets
(a) It is a procedure for which are expected to provide approval/articl came up with situation in banking sector?
considering ecological costs upside to the lenders when the e16437605.ec many schemes 1. Scheme for sustainable
of developmental schemes borrower turns around. e to adress structuring of stressed assets
formulated by the http://indiane these issues. (S4A)
Government. xpress.com/ar Ex - S4A, 5:25 2. Joint Lender"s
(b) It is a scheme of RBI for ticle/business/ etc. Forum
reworking the financial banking-and- 3. Double taxation avoidance
structure of big corporate finance/rbi- agreement (DTAA)
entities facing genuine unveils-new- Select the correct answers
difficulties. scheme-to- using the code given below.
(c) It is a disinvestment plan tackle-bad- (a) 1 and 2 only
of the Government regarding loans-of-big- (b) 2 and 3 only
Central Public Sector firms- (c) 3 only
Undertakings. 2851186/ (d) 1, 2 and 3
(d) It is an important
provision in ‘The Insolvency
and Bankruptcy Code’
recently implemented by the
Government.
15 Environment Consider the following B The Climate & Clean Air Coalition D CAA http://timesof RR The Pune
statements: is the only global effort that unites india.indiatim Municipal
1. Climate and Clean Air governments, civil society and es.com/city/v Corporation
Coalition (CCAC) to Reduce private sector, committed to aranasi/Solid- (PMC) was
Short Lived Climate improving air quality and waste- invited to be a
Pollutants is a unique protecting the climate in next few management- member of
initiative of G2O group of decades by reducing short-lived system-a- Climate and
countries. climate pollutants across sectors. failure-in- Clean Air
2. The CCAC focuses on The Coalition's initial focus is on Varanasi-says- Coalition
methane, black carbon and methane, black carbon, and HFCs. report/articles (CCAC) in
hydrofluorocarbons. The governments of Bangladesh, how/2998095 2013.
Which of the statements Canada, Ghana, Mexico, Sweden 5.cms
given above is/are correct? and the United States, along with http://www.cc
(a) 1 only the United Nations Environment acoalition.org/
(b) 2 only Programme (UNEP), came en/partners
(c) Both 1 and 2 together to initiate the first effort http://www.t
(d) Neither 1 nor 2 to treat these pollutants as a hehindubusin
collective challenge. The coalition essline.com/n
has 53 country partners and 17 ews/national/
International Governmental urgent-steps-
Organizations and 45 Non to-check-4-

13 www.visionias.in ©Vision IAS


governmental organization key-climate-
partners. India is not a partner pollutants-
country however The Energy and can-help-save-
Resources Institute (TERI ) is a 2-million-
partner NGO since 2015. deaths-
annually-
who/article77
96565.ece
16 Geography With reference to ‘Indian B Statement 1 is not correct: The D FCA http://indiane EM The question All India Test Series Test 2134
Ocean Dipole (IOD)’ Indian Ocean Dipole is an irregular xpress.com/ar aims to test Which among the following can
sometimes mentioned in the oscillation of sea-surface ticle/explaine fundamentals suppress rainfall from the
news while forecasting temperatures in which the d/indian- in Geography. Indian Monsoons?
Indian monsoon, which of western Indian Ocean becomes ocean-dipole- IOD and El 1. La-Nina
the following statements alternately warmer and then and-the- Nino are also 2. Negative Indian Ocean
is/are correct? colder than the eastern part of monsoon-the- seen in news Dipole
1. IOD phenomenon is the ocean. joker-in-the- frequently. 3. Active phase of Madden
characterised by a difference Statement 2 is correct: an IOD can forecast-pack/ Julien Oscillation
in sea surface temperature either aggravate or weaken the Select the correct answer using
between tropical Western impact of El Nino on Indian the code given below.
Indian Ocean and tropical monsoon. If there is a positive (a) 1 and 2 only
Eastern Pacific Ocean. IOD, it can bring good rains to (b)2 only
2. An IOD phenomenon can India despite of an El Nino year. (c) 2 and 3 only
influence an El Nino’s impact (d) 1,2 and 3
on the monsoon.
Select the correct answer
using the code given below:
(a) 1 only
(b) 2 only
(c) Both 1 and 2
(d) Neither 1 nor 2
17 Environment If you want to see gharials in B National Chambal Sanctuary, also E CAA NCERT: EN Recently it All India Test Series Tests 2118,
their natural habitat, which called the National Chambal Contemporary was reported 2212, 2352
one of the following is the Gharial Wildlife Sanctuary is a tri- India - II , that the Consider the following
best place to visit? state protected area in northern Chapter -2 population of statements regarding a
(a) Bhitarkanika Mangroves India for the critically endangered Forest and Ghariyals has "Protected Area":
(b) Chambal River gharial (small crocodiles), the red- Wildlife increased in 1. It falls in the tri-point of
(c) Pulicat Lake crowned roof turtle and the Resources. the Chambal Rajasthan, Madhya Pradesh
(d) Deepor Beel endangered Ganges river dolphin. http://timesof Gharial and Uttar Pradesh.
india.indiatim Wildlife 2. It hosts critically endangered
es.com/city/a Sanctuary. species such as Gharial, the
gra/number- Red-crowned Roof Turtle and
14 www.visionias.in ©Vision IAS
of-gharials- the Ganges River Dolphin.
muggers-goes- 3. The headquarter of the
up-in- sanctuary is situated at Morena
chambal/articl in Madhya Pradesh.
eshow/57993 Which of the following
791.cms sanctuaries is best described by
http://www.t the above mentioned
hehindu.com/ statements?
sci- (a) National Chambal Sanctuary
tech/energy- (b) Palpur Kuno Wildlife
and- Sanctuary
environment/i (c) Sanjay Gandhi National Park
nfant- (d) Pench National Park
endangered-
ninja-
turtles/article
18593863.ece
18 Current Consider the following in B The ‘Indian Ocean Naval D CAA http://www.t EN In-depth
Affairs respect of Indian Ocean Symposium’ (IONS) is a voluntary hehindu.com/ knowledge
Naval Symposium (IONS): initiative that seeks to increase news/national about IONS.
1. Inaugural IONS was held in maritime co-operation among /we-will-be- Also, Indian
India in 2015 under the navies of the littoral states of the able-to- ocean has
chairmanship of the Indian Indian Ocean Region by providing liquidate-the- been a focus
Navy. an open and inclusive forum for shortage-of- area for the
2. IONS is a voluntary discussion of regionally relevant women- government in
initiative that seeks to maritime issues. officers-in- the light
increase maritime co- There are 35 members - navies of five-to-six- growing
operation among navies of the IONS which have been years-navy- Chinese
the littoral states of the geographically grouped into the chief/article17 presence in
Indian Ocean Region. following four sub-regions:- South 664481.ece the region.
Which of the above Asian Littorals, West Asian http://pib.nic.i
statements is/are correct? Littorals, East African Littorals, n/newsite/Pri
(a) 1 only South East Asian and Australian ntRelease.asp
(b) 2 only Littorals. x?relid=13433
(c) Both 1 and 2 The inaugural IONS-2008 was held 7
(d) Neither 1 nor 2 in New Delhi, India on 14 Feb 08. http://ions.go
CNS, Indian Navy was designated v.in/
the Chairman IONS for the period
2008-10.
The theme of the IONS-2008 was
“Contemporary Trans-national

15 www.visionias.in ©Vision IAS


Challenges – International
Maritime Connectivities”.
Thus statement 2 is correct and
statement 1 is not correct.
19 Art and The painting of Bodhisattva A The painting is on the back wall of E F AN EM To test basic
Culture Padmapani is one of the the interior hall before the shrine- INTRODUCTIO knowledge of
most famous and oft- antechamber in Cave No. 1 at N TO INDIAN art and
illustrated paintings at Ajanta dating back to the late fifth ART, page 54 architecture
(a) Ajanta century CE. The Boddhisattva is of Ancient
(b) Badami holding a padma (lotus), has large India
(c) Bagh shoulders, and has three bents in
(d) Ellora the body creating a movement in
the picture space. The modelling
is soft. The figure of the
Boddhisattva is wearing a big
crown in which detailed rendering
is visible.
20 Art and Consider the following pairs: A Pair 1 is matched correctly: Chalia D U http://timesof RR To check in
Culture Traditions Communities sahib is the festival of the Sindhis india.indiatim depth
1. Chaliha Sahib Festival – which involves devotees fasting es.com/city/p knowledge
Sindhis for days. une/An- about cultural
2. Nanda Raj Jaat Yatra – Pair 2 is not matched correctly: unusual- heritage of
Gonds The Nanda Devi Jat is celebrated Warkari-at- India.
3. Wari-Warkari - Santhals every year in the Nanda Devi the-
Which of the pairs given temples spread across Garhwal Wari/articlesh
above is/are correctly and Kumaon. The festival ow/52951360.
matched? commences on the day cms
(a) 1 only earmarked for the Devi’s visit to http://timesof
(b) 2 and 3 only her maiti or parents’ home and india.indiatim
(c) 1 and 3 only culminates with the return to her es.com/city/d
(d) None of the above husband’s home. This tradition is elhi/Jhulelal-
beautifully expressed in the folk festival/article
songs of Uttarakhand. show/134353.
Pair 3 is not matched correctly: cms
Pandharpur Wari or Wari (Vari) is
an annual pilgrimage to
Pandharpur - the seat of the god
Vithoba in Maharashtra, in
honour of the deity.
Palakhiscarrying the paduka (foot
prints) of various saints - most
16 www.visionias.in ©Vision IAS
notably Dnyaneshwar and
Tukaram - from the Warkari sect
are taken from their respective
shrines to Pandharpur.
21 Environment Which of the following D No-till farming (also called zero M FA http://www.fa RR http://econo Partly covered in Test 2136.
statements can help in water tillage or direct drilling) is a way of o.org/docrep/ mictimes.india Consider the following pairs:
conservation in agriculture? growing crops or pasture from 003/T0234E/T times.com/ne Types of Farming Description
1. Reduced or zero tillage of year to year without disturbing 0234E04.htm ws/economy/f 1. Mixed farming : different
the land the soil through tillage. No-till is http://www.cr oreign- varieties of crops grown on the
2. Applying gypsum before an agricultural technique which oplife.com/cro trade/pakistan same land simultaneously.
irrigating the field increases the amount of water p- -exporting- 2. Cover cropping : grasses
3. Allowing crop residue to that infiltrates into the soil and inputs/micron 2700-tonnes- planted to provide seasonal soil
remain in the field increases organic matter utrients/the- gypsum-daily- cover on cropland when the
Select the correct answer retention and cycling of nutrients role-of- to- soil would otherwise be bare.
using the code given below: in the soil. gypsum-in- india/articlesh 3. Zero-tillage farming :
(a) 1 and 2 only Gypsum improves the ability of agriculture-5- ow/51457239. growing of crops without
(b) 3 only soil to drain and not become key-benefits- cms disturbing the soil through
(c) 1 and 3 only waterlogged thus increasing you-should- tillage.
(d) 1, 2 and 3 water-use efficiency of crops. know/ Which of the pairs given above
is/are correctly matched?
(a) 3 only
(b) 1 and 2 only
(c) 2 and 3 only
(d) 1, 2 and 3
22 Current Consider the following B Launched by the central E CAA http://pib.nic.i EN Soil health Covered in All India Test Series
Affairs statements: government in February 2015, the n/newsite/Pri card has 2125,2219
The nation-wide ‘Soil Health scheme is tailor-made to issue ntRelease.asp remained With reference to Soil Health
Card Scheme’ aims at ‘Soil card’ to farmers which will x?relid=15944 continuously Card Scheme, consider the
1. expanding the cultivable carry crop-wise recommendations 1 in news due to following Statements:
area under irrigation. of nutrients and fertilizers http://www.t increased 1. It is field-specific detailed
2. enabling the banks to required for the individual farms. hehindu.com/ focus on farm report of soil fertility status .
assess the quantum of loans This is aimed to help farmers to news/national output. 2. It aims to provide farm
to be granted to farmers on improve productivity through /other- insurance based on soil health.
the basis of soil quality. judicious use of inputs. states/award- 3. Every soil health card will be
3. checking the overuse of Thus only statement 3 is correct. for-himachal- renewed every year.
fertilizers in farmlands. for- Which of the statements given
Which of the above implementing- above is/are correct?
statements is/are correct? soil-health- (a) 1 only
(a) 1 and 2 only (b) 3 only card- (b) 1 and 2 only
(c) 2 and 3 only (d) 1, 2 and scheme/articl (c) 2 and 3 only
3 e8506563.ece (d) 1, 2 and 3
17 www.visionias.in ©Vision IAS
18 www.visionias.in ©Vision IAS
23 Science and Consider the following pairs: D Lead is a common impurity found D FA http://www.t EN MSG was in
Technology Commonly used/consumed in cosmetic lip products, such as hehindu.com/l news due to
materials Unwanted or lipsticks, and other cosmetics, ife-and- maggi
controversial chemicals likely such as eye shadows, blushes, style/fashion/ controversy.
to be found in them compact powders, shampoos, and Beauty-
1. Lipstick – Lead body lotions. without-the-
2. Soft drinks – Brominated Brominated vegetable oil is a food beast-organic-
vegetable oils additive sometimes used to keep eco-friendly-
3. Chinese fast food – citrus flavoring from separating and-vegan-
Monosodium glutamate out in some sodas and soft drinks. cosmetics-in-
Which of the pairs given It's banned as a food additive in India/article17
above is/are correctly Europe and Japan. 381697.ece
matched? MSG, is a glutamate, or salt of http://www.t
(a) 1 only glutamic acid, a “non-essential” hehindu.com/
(b) 2 and 3 only amino acid. It is a news/national
(c) 1 and 3 only neurotransmitter - transporting /no-added-
(d) 1, 2 and 3 messages from one nerve cell to msg-claim-
another. Because it is said to did-maggi-
enhance flavours, some scientists in/article7313
believe it “excites nerve endings” 318.ece
and exhibits “neuro-excitatory http://www.t
properties”, that is the ability to hehindu.com/
stimulate neurons. It is commonly todays-
used in chinese fast food. paper/tp-
international/
coke-to-drop-
ingredient-
entirely/articl
e5984124.ece
24 Science and Organic Light Emitting C All the three statements are D FA http://www.a EN OLED being a
Technology Diodes (OLEDs) are used to correct with regard to OLED and bout- new display
create digital display in many they are the disadvatages of LCDs. oled.com/adv technology
devices. What are the Flexible Organic Light Emitting antages-and- has been in
advantages of OLED displays Diodes (FOLED) fabricated on drawbacks.ht news
over Liquid Crystal displays? flexible plastic substates can be ml constantly.
1. OLED displays can be bent or rolled while operating.
fabricated on flexible plastic The manufacturing process of
substrates. OLEDs is different to those of LCD
2. Roll-up displays embedded technology. OLEDs can be printed
in clothing can be made onto almost any substrate with
using OLEDs. inkjet printer technology. That is

19 www.visionias.in ©Vision IAS


3. Transparent displays are why new applications like displays
possible using OLEDs. embedded in clothes or roll-up
Select the correct answer displays are possible.
using the code given below: LCD technology engages a
(a) 1 and 3 only backlight, whereas OLED has no
(b) 2 only backlighting function. Hence it can
(c) 1, 2 and 3 be used for making transparent
(d) None of the above displays.
statements is correct
25 Art and Which of the following is/are A The Sun Temple is situated in D U http://www.t RR To check in
Culture famous for Sun temples? Arasavalli Village in Andhra hehindu.com/ depth
1. Arasavalli Pradesh. Sun temples are not news/national knowledge
2. Amarakantak located in Amarkantak and /andhra- about cultural
3. Omkareshwar Omkareshwar. pradesh/Arasa heritage of
Select the correct answer valli-temple- India.
using the code given below: wears-a-
(a) 1 only festive-
(b) 2 and 3 only look/article17
(c) 1 and 3 only 150240.ece
(d) 1, 2 and 3
26 Polity and Consider the following D Statement 1 is not correct. In M F Laxmikant - EM to check the All India test series - test 2105
Governance statements: election for lok sabha and state Indian Polity, basic (partially covered)
1. In the election for Lok assembly, the first past the post chapter - knowledge Consider the following
Sabha or State Assembly, the system is followed where a "Parliament" statements with regard to
winning candidate must get candidate who polls more votes election system in India:
at least 50 percent of the than any other candidate is 1. India chose first past the
votes polled, to be declared elected. post system because of its
elected. Statement 2 is incorrect. As per cultural, religious and linguistic
2. According to the the constitution, the speaker and diversity.
provisions laid down in the deputy speaker in lok sabha are 2. All the members of Rajya
Constitution of India, in Lok elected among its members. Sabha are elected by
Sabha, the Speaker’s post proportional representation
goes to the majority party system.
and the Deputy Speaker’s to Which of the statements give
the Opposition. above is/are correct?
Which of the statements (a) 1 only
given above is/are correct? (b) 2 only
(a) 1 only (c) Both 1 and 2
(b) 2 only (d) Neither 1 nor 2
(c) Both 1 and 2
(d) Neither 1 nor 2
20 www.visionias.in ©Vision IAS
27 Economics Which of the following B The post-reform period shows the E FA http://www.fi RR To check basic All India test series, test 2112
has/have occurred in India gradual decline in the agriculture rstpost.com/b knowledge of Which of the following is/are
after its liberalization of sector’s contribution to the Indian usiness/25- Indian the effects of New Economic
economic policies in 1991? economy. India’s traditional years-of- economic Policy 1991 on Indian
1. Share of agriculture in GDP occupation, agriculture now liberalisation- development economy?
increased enormously. contributes only about 15% to the a-glimpse-of- 1. Increase in foreign direct
2. Share of India’s exports in GDP, down from 29 percent in indias-growth- investment and forex reserves.
world trade increased. 1991. in-14-charts- 2. Increase in public investment
3. FDI inflows increased. Hence, statement 1 is incorrect. 2877654.html in agricultural infrastructure.
4. India’s foreign exchange Share of India's exports increased 3. Reduction in regional
reserves increased since 1991. disparities.
enormously. Hence, statement 2 is correct. Select the correct answer using
Select the correct answer Before 1991, foreign investment the code given below.
using the codes given below: was negligible. The first year of (a) 1 and 2 only
(a) 1 and 4 only reform saw a total foreign (b) 1 only
(b) 2, 3 and 4 only investment of only $74 million. (c) 2 and 3 only
(c) 2 and 3 only However, investments have (d) 1, 2 and 3
(d) 1, 2, 3 and 4 steadily risen since then, except
for occasional blips between 1997
and 2000 and 2008 and 2012 –
owing to the global economic
slowdown. As of 31 March 2016,
the country has received total FDI
of $371 billion, since 1991. The
year 2008 recorded the highest
FDI inflow of $43.40 billion. The
biggest spurt in inflow was
between 2005 and 2006 –
175.54%. As of March 2016, India
has attracted $10.55 billion worth
of FDI. Hence, statement 3 is
correct.
It was India’s dismal state of forex
reserves that forced the
government to bring in economic
reforms. Now, 25 years later,
forex reserves are at a record
high. In 1991, it stood at just $5.8
billion. As of 24 June, the
country’s forex reserves are at
$360.8 billion. Usually, import

21 www.visionias.in ©Vision IAS


coverage of 7-8 months is
considered sufficient. The biggest
jump in reserves was witnessed
between 2007 and 2008 when the
kitty bulged 55% to hit $309.2
billion. Hence, sttement 4 is
correct.
28 Science and What is the application of C Somatic-cell nuclear transfer M F http://www.t EN 20 years of
Technology Somatic Cell Nuclear Transfer involves removing the nucleus, hehindu.com/ cloning first
Technology? (a) Production which contains the DNA, and todays- sheep - Dolly
of biolarvicides implanting it into an unfertilised paper/tp-in-
(b) Manufacture of egg, whose nucleus has been school/Creatin
biodegradable plastics removed. It is used in g-genetic-
(c) Reproductive cloning of reproductive cloning of copies/article
animals organisms. 14478059.ece
(d) Production of organisms
free of diseases
29 Current Consider the following C (NPCI) is an umbrella organization E CAA http://pib.nic.i EN Digital Covered in PT 365 Economy
Affairs statements: for all retail payments system in n/newsite/Pri payment has and All India Test Series:
1. National Payment India. It was set up with the ntRelease.asp continuously 2121,2215,2239,2363
Corporation of India (NPCI) guidance and support of the x?relid=16033 remained in With reference to National
helps in promoting the Reserve Bank of India (RBI) and 9 nws and NPCI Payments Corporation of India
financial inclusion in the Indian Banks’ Association (IBA). http://www.t is one of the (NPCI), consider the following
country. The core objective was to hehindu.com/ most statements:
2. NPCI has launched RuPay, consolidate and integrate the business/Econ important 1. It is an umbrella organization
a card payment scheme. multiple systems with varying omy/smoothe step in this for all retail payments system
Which of the statements service levels into nation-wide n-bhim- regard. in India.
given above is/are correct? uniform and standard business refund- 2. It has been incorporated
(a) 1 only process for all retail payment process-it- under the Companies Act,
(b) 2 only systems. The other objective was minister-to- 1956.
(c) Both 1 and 2 to facilitate an affordable npci/article18 3. RuPay card payment scheme
(d) Neither 1 nor 2 payment mechanism to benefit 185655.ece has been launched by the NPCI.
the common man across the http://www.n Which of the statements given
country and help financial pci.org.in/ above is/are correct?
inclusion. Thus statement 1 is (a) 1 and 2 only
correct. (b) 1 only
It offers many products and (c) 2 and 3 only
services which include RuPay card, (d) 1, 2 and 3
BHIM app, Unified payment
interface among others. Thus
statement 2 is correct.
22 www.visionias.in ©Vision IAS
23 www.visionias.in ©Vision IAS
30 Environment The term ‘M-STrIPES’ is B Monitoring System for Tigers’- M CAA http://www.n EN Official launch
sometimes seen in the news Intensive Protection and ewindianexpr of M-STrIPES
in the context of Ecological Status is a software ess.com/state (Monitoring
(a) Captive breeding of Wild monitoring system launched by s/odisha/2017 System For
Fauna the Indian Government in 2010 in /feb/08/m- Tigers-
(b) Maintenance of Tiger some tiger reserves to reduce stripes-to- Intensive
Reserves vulnerability of Tigers. The system monitor-str- Protection
(c) Indigenous Satellite would enable field managers to from-april- and Ecological
Navigation System assist intensity and spatial 1568257.html Status)
(d) Security of National coverage of patrols in a http://timesof software-
Highways geographic information system india.indiatim enabled
(GIS) domain. es.com/city/d mobile app at
ehradun/mobi Corbett Tiger
le-app-to- Reserve.
monitor-tiger-
reserves-
launched/artic
leshow/56896
945.cms
31 Economics What is/are the most likely A GST is one indirect tax for the M CAA http://www.g EN Consistently in Covered in PT365 Economy
advantages of implementing whole nation, which will make stindia.com/a news and VisionIAS test series - 2130
‘Goods and Services Tax India one unified common bout/ Which of the following can be
(GST)’? 1. It will replace market. achieved with the
multiple taxes collected by Hence, statement 1 is correct. implementation of the Goods
multiple authorities and will There is no information which and Services Tax Act (GST)?
thus create a single market says GST will drastically reduce 1. Common marketplace for
in India. the CAD of india. Neither it is true direct taxes
2. It will drastically reduce that it will enable India to take 2. Elimination of distortions in
the ‘Current Account Deficit’ over China. intrastate trade
of India and will enable it to Hence, statements 2 and 3 are 3. Boost investment and
increase its foreign exchange not correct. growth
reserves. Select the correct answer using
3. It will enormously increase the code given below.
the growth and size of (a) 1 only
economy of India and will (b) 1 and 2 only
enable it to overtake China in (c) 2 and 3 only
the near future. (d) 1, 2 and 3
Select the correct answer
using the code given below:
(a) 1 only (b) 2 and 3 only
(c) 1 and 3 only (d) 1, 2 and 3

24 www.visionias.in ©Vision IAS


32 Current 'Broad-based Trade and A The FTA talks with EU have been E CA http://pib.nic.i EN FTA Covered in PT 365 Updation -
Affairs Investment Agreement deadlocked since 2013 after 16 n/newsite/Pri negotiations International
(BTIA)’ is sometimes seen in rounds of negotiations. Last year ntRelease.asp with EU has
the news in the context of EU raised its concern over what it x?relid=16013 remained
negotiations held between called India’s “unilateral 1 continuously
India and termination” of existing Bilateral http://www.t in news.
(a) European Union Investment Treaties (BITs) with “a hehindu.com/
(b) Gulf Cooperation Council significant number of” EU- business/Indu
(c) Organization for member countries. stry/India-EU-
Economic Cooperation and aim-to-break-
Development Free-Trade-
(d) Shanghai Cooperation Agreement-
Organization impasse/articl
e14378438.ec
e
33 Current Consider the following A The TFA is the WTO’s first-ever M CA http://pib.nic.i EN TFA has Covered in PT 365 Economy
Affairs statements: multilateral accord that aims to n/newsite/Pri remained and PT 365 Updation -
1. India has ratified to Trade simplify customs regulations for ntRelease.asp regularly in Economy All India test series -
Facilitation Agreement (TFA) the cross-border movement of x?relid=15999 news for last 3 partially covered in test 2124,
of WTO. goods. It was outcome of WTO’s 2 years. 2121 and 2358
2. TFA is a part of WTO’s Bali 9th Bali (Indonesia) ministerial http://www.t Doha round of WTO covers
Ministerial Package of 2013. package of 2013. India ratified it hehindu.com/ negotiations on which of the
3. TFA came into force in in April 2016. It came into force in business/Econ following?
January 2016. February 2017. Thus statement 3 omy/india- 1. Agriculture
Which of the statements is not correct. tables-legally- 2. Services
given above is/are correct? vetted- 3. Non-Agricultural Market
(a) 1 and 2 only proposal-on- Access 4. Trade Facilitation
(b) 1 and 3 only global- Select the correct answer using
(c) 2 and 3 only services-pact- the code given below.
(d) 1, 2 and 3 at- (a) 1 and 3 only (b) 1 and 4 only
wto/article17 (c) 2, 3 and 4 only
355734.ece (d) 1, 2, 3 and 4
34 Current What is the importance of C India and Iran signed a series of E CA http://pib.nic.i EN Chabahar port Covered in PT365 International
Affairs developing Chabahar Port by agreements that allowed New n/newsite/mb has remained Relations . All India test series,
India? Delhi to use the port of Chabahar Erel.aspx?relid continuously test 2123 (covered in
(a) India’s trade with African to access Central Asia and =147456 in news. explanation).
countries will enormously Afghanistan without depending http://www.t Chabahar Port is associated
increase. upon Pakistan. hehindu.com/ with which of the following
(b) India’s relations with oil- news/national locations?
producing Arab countries will /India-to- 1. Gulf of Oman
be strengthened. develop-Irans- 2. Persian Gulf
25 www.visionias.in ©Vision IAS
(c) India will not depend on Chabahar- 3. Arabian Sea
Pakistan for access to port/article14 Select the correct answer using
Afghanistan and Central Asia. 336893.ece the code given below. (a) 1 only
(d) Pakistan will facilitate and (b) 2 and 3 only
protect the installation of a (c) 1 and 3 only
gas pipeline between Iraq (d) 1, 2 and 3
and India.
35 Current In India, it is legally D The Indian Computer Emergency D CAA http://pib.nic.i RR Cyber security
Affairs mandatory for which of the Response Team shall serve as the n/newsite/pri regularly in
following to report on cyber national agency for performing ntrelease.aspx news.
security incidents? the following functions in the area ?relid=158620
1. Service providers of cyber security,– http://lawmin.
2. Data centres (a) collection, analysis and nic.in/ld/P-
3. Body corporate dissemination of information on ACT/2000/The
Select the correct answer cyber incidents; %20Informati
using the code given below: (b) forecast and alerts of cyber on%20Technol
(a) 1 only security incidents; ogy%20Act,%2
(b) 1 and 2 only (c) emergency measures for 02000.pdf
(c) 3 only handling cyber security incidents;
(d) 1, 2 and 3 (d) coordination of cyber incidents
response activities;
(e) issue guidelines, advisories,
vulnerability notes and white
papers relating to information
security practices, procedures,
prevention, response and
reporting of cyber incidents;
(f) such other functions relating to
cyber security as may be
prescribed.
Any service provider,
intermediaries, data centres,
body corporate or person who
fails to provide the information
called for or comply with the
above section shall be punishable
with imprisonment for a term
which may extend to one year or
with fine which may extend to
one lakh rupees or with both.

26 www.visionias.in ©Vision IAS


36 Polity and Right to vote and to be D According to Supreme Court D F EM to check the
Governance elected in India is a judgment Jyoti Basu vs Debi Union of India basic
(a) Fundamental Right Ghosal - A right to elect, vs. Association knowledge
(b) Natural Right fundamental though it is to for
(c) Constitutional Right democracy, is, anomalously Democratic
(d) Legal Right neither a fundamental right nor a Reforms and
Common Law Right. It is a Anr. (2002)
statutory right. So is the right to 5SCC 294 and
be elected, and the right to People’s
dispute an election. Union for Civil
***NOTE - As per NCERT Liberties vs.
One of the important decisions of Union of India
the framers of the Indian (2003)
Constitution was to guarantee
every adult citizen in India, the ****Article
right to vote i.e. universal adult 326 of the
franchise. The article 326 of the constitution &
Constitution provides for the right NCERT - 11,
to vote and to be elected in India. Indian
What is true of the right to vote is constitution at
also true of right to contest work, chapter
election. All citizens have the right 3, page 66
to stand for election and become
the representative of the people.
37 Current What is the purpose of B eLISA is a spectacular plan of E CA http://www.t EN Gravitationals Covered in PT 365 SnT and All
Affairs ‘evolved Laser setting into space three hehindu.com/ waves have India Test Series 2115, 2209, 2360
Interferometer Space spacecraft, a mother and two sci- been regularly Which of the following pairs
Antenna (eLISA)’ project? daughter spacecraft, which will fly tech/science/S in news. is/are correctly matched?
(a) To detect neutrinos in a triangular formation, trailing pace- Spacecraft/ Mission : Purpose
(b) To detect gravitational the earth in its orbit around the antennae-to- 1. New Horizons : to map and
waves sun at a distance of over 50 probe- investigate Mercury
(c) To detect the million km. Laser interferometers gravitational- 2. Juno : to understand origin
effectiveness of missile will accurately measure changes waves/article1 and evolution of Jupiter
defence system in the distance between these 4433813.ece 3. LISA Pathfinder : to
(d) To study the effect of cubes. If they should be affected demonstrate the technology
solar flares on our by a gravitational wave, the needed to build a space-based
communication systems minute changes in this distance gravitational wave observatory
are measure by the Select the correct answer using
interferometer. the code given below.
(a) 1 only (b) 2 and 3 only
(c) 1 and 3 only (d) 1, 2 and 3

27 www.visionias.in ©Vision IAS


38 Current What is the purpose of A The Ministry of HRD recently M CAA http://pib.nic.i EN Many Covered in PT 365 Social and
Affairs ‘Vidyanjali Yojana’? launched the Vidyanjali scheme n/newsite/Pri schemes have All India Test Series
1. To enable the famous aimed at boosting the education ntRelease.asp been lunched 2114,2208,2238,2364
foreign educational system by delivering volunteer x?relid=14627 by the The recently launched
institutions to open their teachers to government schools. 3 government Vidyanjali scheme aims at
campuses in India. It will not replace the regular and http://www.t to improve (a) boosting the education
2. To increase the quality of professionally qualified teachers hehindu.com/ the learning system by delivering volunteer
education provided in in the government schools. news/cities/Vi outcome and teachers to government
government schools by The volunteer's responsibility is jayawada/Vidy quality of the schools.
taking help from the private towards overall development of anjali- education. (b) providing support to young
sector and the community. the child, not academics. launched/artic entrepreneurs by coordinated
3. To encourage voluntary The volunteer service will be used le14427857.ec delivery of various
monetary contributions from in developing skills like public e entrepreneurship programs.
private individuals and speaking, creative writing, (c) introducing compulsory
organizations so as to counseling, music and dance. certification for teachers in
improve the infrastructure government and private
facilities for primary and schools
secondary schools. (d) none of the above
Select the correct answer
using the code given below:
(a) 2 only (b) 3 only
(c) 1 and 2 only
(d) 2 and 3 only
39 Current What is the aim of the B With a view to uplift rural India, E CA http://www.t EN A lot of focus Covered in PT365 - Govt
Affairs programme ‘Unnat Bharat the Unnat Bharat Abhiyan hehindu.com/ in on schemes & All India Test Series
Abhiyan’? programme is launched in news/cities/T grassroot 2125,2219,2366:
(a) Achieving 100% literacy collaboration with the Indian hiruvananthap innovation. With reference to Unnat Bharat
by promoting collaboration Institutes of Technology (IIT) and uram/this- The scheme Abhiyan, consider the following
between voluntary the National Institutes of bureaucrat- was in news in statements:
organizations and Technology (NITs) across the also- this regard. 1. It aims to support smart
government’s education country. heals/article1 cities project and urban
system and local The programme aims to connect 7413890.ece development.
communities. institutions of higher education, http://pib.nic.i 2. It connects IITs and NITs with
(b) Connecting institutions of including Indian Institutes of n/newsite/Pri local communities to address
higher education with local Technology (IITs), National ntRelease.asp the development challenges.
communities to address Institutes of Technology (NITs) x?relid=16079 Which of the statements given
development challenges and Indian Institutes of Science 0 above is/are correct?
through appropriate Education & Research (IISERs) etc. (a) 1 only
technologies. with local communities to address (b) 2 only
(c) Strengthening India’s the development challenges (c) Both 1 and 2
scientific research through appropriate technologies. (d) Neither 1 nor 2

28 www.visionias.in ©Vision IAS


institutions in order to make
India a scientific and
technological power.
(d) Developing human capital
by allocating special funds
for health care and
education of rural and urban
poor, and organizing skill
development programmes
and vocational training for
them.
40 Polity and Consider the following D Only statement 3 is correct. E FA Laxmikant - EM to check the
Governance statements: 1. The Election Election Commission of India is a 3 Indian Polity, basic
Commission of India is a five- member body and it is the chapter - knowledge
member body. election commission which "Election
2. Union Ministry of Home decides the election schedule for Commission"
Affairs decides the election both general elections and bye-
schedule for the conduct of elections.
both general elections and It is also the function of election
bye-elections. commission to resolve the
3. Election Commission disputes relating to splits/mergers
resolves the disputes relating of recognized political parties.
to splits/mergers of
recognized political parties.
Which of the statements
given above is/are correct?
(a) 1 and 2 only (b) 2 only
(c) 2 and 3 only (d) 3 only
41 Environment In India, if a species of A The Act provides for the E FA NCERT: EM Schedule I of All India Test Series - Tests
tortoise is declared protection of wild animals, birds Contemporary the Wildlife 2134, 2127 , Q. With reference
protected under Schedule I and plants; and for matters India - II , (Protection) to the black-necked crane,
of the Wildlife (Protection) connected therewith or ancillary Chapter -2 Act, 1972 is consider the following
Act, 1972, what does it or incidental thereto. It extends to Forest and often seen in statements: 1. The high altitude
imply? the whole of India, except the Wildlife news. wetlands in the Tibetan plateau
(a) It enjoys the same level of State of Jammu and Kashmir Resources. are the main breeding ground
protection as the tiger. which has its own wildlife act. It of the species.
(b) It no longer exists in the has six schedules which give 2. It is given legal protection
wild, a few individuals are varying degrees of protection. under Schedule I of the Wildlife
under captive protection; Schedule I and part II of Schedule (Protection) Act, 1972.
and not it is impossible to II provide absolute protection - Which of the statements given
prevent its extinction. offences under these are above is/are correct?
29 www.visionias.in ©Vision IAS
(c) It is endemic to a prescribed the highest penalties. (a) 1 only (b) 2 only
particular region of India. Species listed in Schedule III and (c) Both 1 and 2
(d) Both (b) and (c) stated Schedule IV are also protected, (d) Neither 1 nor 2
above are correct in this but the penalties are much lower. Which of the following
context. Schedule V includes the animals statements is/are correct
which may be hunted. regarding Wildlife Protection
act, 1972? 1. Schedule V of the
Act contains vermins which can
be hunted.
2. As per the act, only the
central government can declare
an animal as vermin.
3. The Act covers only animals
and not plants.
Select the correct answer using
the code given below.
(a) 1 only (b) 1 and 2 only
(c) 2 and 3 only (d) 1, 2 and 3
42 Polity and In India, Judicial Review A Judicial review is the power of M F Laxmikanth EM to test the
Governance implies judiciary to examine the Ch-30 High basic concept
(a) the power of the Judiciary constitutionality of legislative Court
to pronounce upon the enactments and executive orders
constitutionality of laws and of both the Central and state
executive orders. governments. On examination, if
(b) the power of the Judiciary they are found to be violative of
to question the wisdom of the Constitution (ultra-vires), they
the laws enacted by the can be declared as illegal,
Legislatures. unconstitutional and invalid (null
(c) the power of the Judiciary and viod). Consequently, they
to review all the legislative cannot be enforced by the
enactments before they are government. The constitutional
assented to by the President. validity of a legislative enactment
(d) the power of the Judiciary or an executive order can be
to review its own challenged in a high court or
judgements given earlier in supreme court on the following
similar or different cases. three grounds: (a) it infringes the
fundamental rights (Part III), (b) it
is outside the competence of the
authority which has framed it, and
(c) it is repugant to the
constitutional provisions.

30 www.visionias.in ©Vision IAS


31 www.visionias.in ©Vision IAS
43 Modern With reference to Indian C Second Round Table Conference E F NCERT RR To check the
History freedom struggle, consider took place on 7th September chapter 13 basic
the following events: 1931. class 12 knowledge of
1. Mutiny in Royal Indian The Quit India Movement, also themes in all the
Navy known as the August Movement indian history- important
2. Quit Indian Movement was a Civil Disobedience 3 historical
launched Movement launched by Gandhi JI events.
3. Second Round Table on 8th August 1942.
Conference Royal Indian Navy (RIN) mutiny
What is the correct was a rebellion launched on
chronological sequence of February 18, 1946, by seamen on
the above events? the HMIS Talwar. Hence correct
(a) 1-2-3 chronological sequence of the
(b) 2-1-3 above events are 3-2-1
(c) 3-2-1
(d) 3-1-2
44 Economics Consider the following D Service tax, personal income tax M CAA Economic RM Economic VisionIAS 2017 Open test 4-
statements: and corporation tax have been survey 2014- survey has 2375 Q. With respect to Indian
1. Tax revenue as a percent reduced in the recent past to 15 discussed this economy, consider the
of GDP of India has steadily boost aggregate demand, so as to issue in detail. following statements: 1. The
increased in the last decade. protect the economy from global current account balance has
2. Fiscal deficit as a percent recession. remained negative in the last
of GDP of India has steadily Excise duty rates have been three years. 2. The fiscal deficit
increased in the last decade. reduced in the recent past to has steadily declined over the
Which of the statements boost aggregate demand, so as to last three years. Which of the
given above is/are correct? protect the economy from global statements given above is/are
(a) 1 only recession hence tax revenue as a correct? (a) 1 only (b) 2 only (c)
(b) 2 only percent of GDP of India has Both 1 and 2 (d) Neither 1 nor 2
(c) Both 1 and 2 steadily not increase in the last
(d) Neither 1 nor 2 decade . Hence statement 1 is not
correct.
Fiscal deficit as a percent of GDP
of India has not steadily increased
in the last decade. Hence
statement 2 is not correct.
45 Current Recently there was a B An environment ministry’s expert M CA http://www.t EN Important
Affairs proposal to translocate some committee approved Kuno Palpur hehindu.com/ news
of the lions from their in Madhya Pradesh as the second news/national regarding lion
habitat in Gujarat to which home for Asiatic lions found only /other- conservation
one of the following sites? in Gir national park. However, the states/M.P.-
(a) Corbett National Park Gujarat government will not share seeks-
32 www.visionias.in ©Vision IAS
(b) Kuno Palpur Wildlife lions unless 33 studies as %E2%80%98i
Sanctuary mandated by international wildlife mmediate%E2
(c) Mudumalai Wildlife watchdog IUCN is completed %80%99-
Sanctuary transfer-of-
(d) Sariska National Park Asiatic-lions-
from-
Gujarat/article
14477032.ece
46 Polity and Which of the following are B When the President’s Rule is M F Laxmikanth EM to test the
Governance not necessarily the imposed in a state, the President Ch-16 basic
consequences of the dismisses the state council of Emergency knowledge
proclamation of the ministers headed by the chief Provisions
President’s rule in a State? minister. The state governor, on
1. Dissolution of the State behalf of the President, carries on
Legislative Assembly the state administration with the
2. Removal of the Council of help of the chief secretary of the
Ministers in the State state or the advisors appointed by
3. Dissolution of the local the President. This is the reason
bodies why a proclamation under Article
Select the correct answer 356 is popularly known as the
using the code given below: imposition of ‘President’s Rule’ in
(a) 1 and 2 only a state. Further, the President
(b) 1 and 3 only either suspends or dissolves the
(c) 2 and 3 only state legislative assembly.The
(d) 1, 2 and 3 Parliament passes the state
legislative bills and the state
budget. The President's Rule
doesnot lead to dissolution of
local bodies.
Hence, the answer is 1 and 3 only.
47 Polity and Which of the following are C RIGHT AGAINST EXPLOITATION E F Laxmikanth - EM to test the All India test series - test 2114
Governance envisaged by the Right includes Article 23 & 24 of the Ch-7 factual and Right against exploitation
against Exploitation in the Indian Constitution. Fundamental basic guaranteed by the Constitution
Constitution of India? Article 23 prohibits traffic in Rights knowledge prohibits which of the following
1. Prohibition of traffic in human beings, begar (forced activities?
human beings and forced labour) and other similar forms of (a) Exploitation of religious
labour forced labour. Any contravention minorities.
2. Abolition of untouchability of this provision shall be an (b) Exploitation of Scheduled
3. Protection of the interests offence punishable in accordance Castes and Scheduled Tribes
of minorities with law. This right is available to (c) Trafficking in human beings
4. Prohibition of both citizens and non-citizens. It and forced labour
33 www.visionias.in ©Vision IAS
employment of children in protects the individual not only (d) Forced migration due to
factories and mines against the State but also against displacement and development
Select the correct answer private persons. activities
using the code given below: Article 24 prohibits the
(a) 1, 2 and 4 only employment of children below
(b) 2, 3 and 4 only the age of 14 years in any factory,
(c) 1 and 4 only mine or other hazardous
(d) 1, 2, 3 and 4 activities like construction work
or railway. But it does not prohibit
their employment in any harmless
or innocent work.
48 Geography Which of the following is A Great Nicobar is the M FA Atlas EM Important All India Test Series - Test 2118
geographically closest to southernmost and largest of the http://www.li from the view (partially covered) Which one
Great Nicobar? Nicobar Islands of India, north of vemint.com/L point of of the following pairs of islands
(a) Sumatra Sumatra. The island of Sumatra is eisure/bCr0Gp India's is separated from each other by
(b) Borneo located 180 km (110 mi) to the LRbAL603xnOj territorial the "Eight Degree Channel?
(c) Java south of Great Nicobar. VT5J/Fear- extent. (a) Andman and Nicobar
(d) Sri Lanka and-survival- (b) Nicobar and Sumatra
in-Great- (c) Maldives and Lakshadweep
Nicobar.html (d) Sumatra and Java
49 Polity and Out of the following C The Parliamentary system is also M F Laxmikanth EM to test the
Governance statements, choose the one known as the ‘Westminster’ Ch-Salient basic clarity
that brings out the principle model of government, responsible Features of
underlying the Cabinet form government and cabinet Indian
of Government: government. The Constitution Constitution.
(a) An arrangement for establishes the parliamentary
minimizing the criticism system not only at the Centre but
against the Government also in the states. The features of
whose responsibilities are parliamentary government in
complex and hard to carry India are: (a) Presence of nominal
out the satisfaction of all. and real executives; (b) Majority
(b) A mechanism for party rule, (c) Collective
speeding up the activities of responsibility of the executive to
the Government whose the legislature, (d) Membership of
responsibilities are the ministers in the legislature, (e)
increasing day by day. Leadership of the prime minister
(c) A mechanism of or the chief minister, (f)
parliamentary democracy for Dissolution of the lower House
ensuring collective (Lok Sabha or Assembly).
responsibility of the
Government to the people.
34 www.visionias.in ©Vision IAS
(d) A device for
strengthening the hands of
the head of the Government
whose hold over the people
is in a state of decline.
50 Polity and Which one of the following is D The Constitution of India E F Laxmikanth- EM to test the VisionIAS Test Series - Tests
Governance not a feature to Indian establishes a federal system of CH-Salient basic clarity 2104 & 2203
federalism? government. It contains all the Features of In the context of Indian
(a) There is an independent usual features of a federation, the federation, consider the
judiciary in India. viz., two government, division of Constitution following statements:
(b) Powers have been clearly powers, written Constitution, 1. States have no right to
divided between the Centre super-macy of Constitution, secede from the federation.
and the States. rigidity of Constitution, 2. It is the result of an
(c) The federating units have independent judiciary and agreement among the states.
been given unequal bicameralism. Which of the statements given
representation in the Rajya Moreover, the term ‘Federation’ above is/are correct?
Sabha. has nowhere been used in the (a) 1 only
(d) It is the result of an Constitution. Article 1, on the (b) 2 only
agreement among the other hand, describes India as a (c) Both 1 and 2
federating units. ‘Union of States’ which implies (d) Neither 1 nor 2
two things: one, Indian
Federation is not the result of an
agreement by the states; and
two, no state has the right to
secede from the federation.
51 Modern The object of the Butler D The Report of the Butler M F spectrum EM To check the All India Open test 2(2226),All
History Committee of 1927 was to Committee on the relations page no.321 basic India test series - 2110
(a) Define the jurisdiction of between the Indian States and knowledge. Harcourt Butler Committee was
the Central and Provincial British India . The Committee appointed by the British
Governments. advise that the Viceroy (instead of Government on December 16,
(b) Define the powers of the the Governor-General in Council) 1927 to
Secretary of State for India. should represent the Crown in all (a) curb the left wing
(c) Impose censorship on dealings with the States. This movement.
national press. proposed change is comparable (b) grant autonomy to British
(d) Improve the relationship with the new Imperial administered province.
between the Government of arrangement by which the (c) strengthen the relationship
India and the Indian States. Governor-General of a Dominion of princely states with the
is High Commissioner for the Crown.
Dominion's Protectorates. The (d) modernize Indian
Committee sympathize with the agriculture.
fear of the Native Princes that (2) Consider the following pairs:
35 www.visionias.in ©Vision IAS
their States might pass without Committee Objective
their consent under a new 1. Butler committee : to clarify
Government in British India the relationship between the
responsible to an Indian British crown and the Princely
Legislature, and they plainly states.
express the opinion that no such 2. Hunter Committee : to
transference should be made report on development of local
without the consent of the self government
Princes. 3. Hartog Committee : to look
into the Punjab wrongs Which
of the pairs given above is/are
correctly matched?
(a) 1 only (b) 1 and 3 only
(c) 2 and 3 only (d) 1, 2 and 3
52 Current The term ‘Domestic Content A ‘Domestic Content Requirement’, E F http://econo EN WTO ruling Covered in 2017 Vision IAS
Affairs Requirement’ is sometimes recently in news, is related to mictimes.india against India. Open test 1-2225
seen in the news with development of solar power times.com/ind Which of the following is not
reference to production. ustry/energy/ correct about National Solar
(a) Developing solar power The World Trade Organization had power/remov Mission?
production in our country ruled against India's domestic al-of- (a) The Mission has the target
(b) Granting licenses to content policy for solar cells and domestic- of achieving 1,00,000 MW solar
foreign T.V. channels in our modules; India appealed the content- power by 2022.
country ruling in a bid to keep DCR rules in requirement- (b) WTO has allowed India to
(c) Exporting our food place for government in-solar-not-a- mandate Domestic Content
products to other countries procurement. The case against big- Requirement in achieving the
(d) Permitting foreign India was originally filed in 2013, blow/articlesh target under the mission.
educational institutions to following the announcement of ow/54412591. (c) It is one of the eight
set up their campuses in our DCR in JNNSM Phase II policy and cms missions of National Action
country after India decided to file an anti- Plan on Climate Change
dumping case against the U.S., (NAPCC).
China, Malaysia and Taiwan. (d) Ministry of New and
Renewable Energy is the
implementing agency for the
National Solar Mission.
53 Current Consider the following D Nuclear Security Summits are an D U http://indiane RR The fourth
Affairs statements: United States-led initiative and xpress.com/ar edition of
1. The Nuclear Security are not under the aegis of the ticle/explaine Nuclear
Summits are periodically United Nations. d/nuclear- Security
held under the aegis of the The International Panel on Fissile security- Summit (NSS)
United Nations. Materials (IPFM) was founded in summit-how- was held in
2. The International Panel on January 2006 and is an to- Washington,
36 www.visionias.in ©Vision IAS
Fissile Materials is an organ independent group of arms- understand- United States
of International Atomic control and nonproliferation what-it-does/ last year.
Energy Agency. experts (and not an organ of http://fissilem
Which of the statements International Atomic Energy aterials.org/ip
given above is/are correct? Agency) from both nuclear fm/about.htm
(a) 1 only weapon and non-nuclear weapon l
(b) 2 only states. Its members include
(c) Both 1 and 2 nuclear experts from seventeen
(d) Neither 1 nor 2 countries: Brazil, Canada, China,
France, Germany, India, Iran,
Japan, Mexico, Norway, Pakistan,
South Korea, Russia, South Africa,
Sweden, the United Kingdom, and
the United States.
54 Current Who among the following C NPS (National Pension System) is M FCA https://india.g EN Recent
Affairs can join the National Pension a defined contribution based ov.in/spotlight changes in the
System (NPS)? Pension Scheme launched by /national- scheme.
(a) Resident Indian citizens Government of India. Eligibility- pension-
only - NPS is applicable to all new system-
(b) Persons of age from 21 to employees of Central Government retirement-
55 only service (except Armed Forces) and plan-all
(c) All State Government Central Autonomous Bodies http://www.t
employees joining the joining Government service on or hehindu.com/
services after the state of after 1st January 2004. business/Econ
notification by the respective - NPS is applicable to all the omy/NRIs-
State Governments employees of State Governments, can-now-
(d) All Central Government State Autonomous Bodies joining apply-online-
employees including those of services after the date of for-National-
Armed Forces joining the notification by the respective Pension-
services on or after 1st April, State Governments. Scheme/articl
2004 - All citizens of India between the e14428129.ec
age of 18 and 60 years as on the e
date of submission of his / her
application to Point of Presence
(POP) / Point of Presence-Service
Provider (POP-SP) can join NPS.
Recently, NRIs were also allowed
to open National Pension Scheme
(NPS) accounts online.

37 www.visionias.in ©Vision IAS


55 Geography With reference to river B Statement 1 is not correct. The M FCA EM River Teesta PT 365 Updation Material, Page
Teesta, consider the Teesta River originates from the has been 16. Eliminating statement 3,
following statements: Pahunri (or Teesta Kangse) glacier often seen in one could arrive at the correct
1. The source of river Teesta and flows southward through the news with answer.
is the same as that of Sikkim Himalaya. where the respect to
Brahmaputra but it flows Rangpo River joins, and where it India-
through Sikkim. forms the border between Sikkim Bangladesh
2. River Rangeet originates in and West Bengal up to Teesta international
Sikkim and it is a tributary of Bazaar. Just before the Teesta relations. Its
river Teesta. Bridge, where the roads from waters are still
3. River Teesta flows into Bay Kalimpong and Darjeeling join, the disputed
of Bengal on the border of river is met by its main tributary, between the
India and Bangladesh. the Rangeet River. Hence two nations.
Which of the statements Statement 2 is correct.
given above is/are correct? It changes course southwards
(a) 1 and 3 only flowing into West Bengal. The
(b) 2 only river then goes merging up with
(c) 2 and 3 only the Brahmaputra River after it
(d) 1, 2 and 3 bifurcates the city of Jalpaiguri
and flows just touching Cooch
Behar district at Mekhliganj and
moves to Fulchori in Bangladesh.
Thus it does not flow directly into
the Bay of Bengal. Hence
statement 3 is not correct.
56 Current Consider the following C Zika virus is primarily transmitted M FCA Atlas EN Last year, Covered in june 2016 current
Affairs statements: to people through the bite of an World Health affairs All India Test series, test
1. In tropical regions, Zika infected mosquito from the Aedes Organization 2114 and 2360 (partially
virus disease is transmitted genus, mainly Aedes aegypti in (WHO) covered)
by the same mosquito that tropical regions. This is the same confirmed Consider the following
transmits dengue. mosquito that transmits dengue, India’s first statements about Zika virus
2. Sexual transmission of Zika chikungunya and yellow fever. three cases of disease: 1. It is caused by a
virus disease is possible. Although Zika virus is primarily Zika virus. virus transmitted by mosquito.
Which of the statements transmitted through mosquitoes, 2. Presently there is no vaccine
given above is/are correct? it can also be sexually transmitted available for Zika virus.
(a) 1 only as well. 3. Zika can be passed from a
(b) 2 only pregnant woman to her foetus.
(c) Both 1 and 2 Which of the statements given
(d) Neither 1 nor 2 above is/are correct?
(a) 1 only (b) 1 and 2 only
(c) 2 and 3 only (d) 1, 2 and 3

38 www.visionias.in ©Vision IAS


57 Current Consider the following A In 2010, a certification from the D U http://indiane RR AGMARK has
Affairs statements: Bureau of Indian Standards (BIS) xpress.com/ar been in use to
1. The Standard Mark of has been made mandatory for all ticle/india/zik promote
Bureau of Indian Standards types of automotive tyres and a-virus-all- agricultural
(BIS) is mandatory for tubes. you-need-to- produce.
automotive tyres and tubes. AGMARK is a certification mark know-
2. AGMARK is a quality employed on agricultural products 4676548/
Certification Mark issues by in India, assuring that they https://www.
the Food and Agriculture conform to a set of standards nhp.gov.in/Zik
Organisation (FAO). approved by the Directorate of a-virus_pg
Which of the statements Marketing and Inspection, an http://www.w
given above is/are correct? agency of the Government of ho.int/mediac
(a) 1 only India. The AGMARK is legally entre/factshe
(b) 2 only enforced in India by the ets/zika/en/
(c) Both 1 and 2 Agricultural Produce (Grading and
(d) Neither 1 nor 2 Marking) Act of 1937 (and
ammended in 1986).
58 Economics What is/are the C National Agriculture Market E CAA http://www.t EN One of the Covered in PT 365 Economy All
advantage/advantages of (NAM) is a pan-India electronic hehindu.com/ important India test series, test 2112,
implementing the ‘National trading portal which networks the news/cities/ch reforms in 2357
Agriculture Market’ scheme? existing APMC mandis to create a ennai/BIS- agricutural With reference to the National
1. It is a pan-India electronic unified national market for certification- sector. Agricultural Market (NAM),
trading portal for agricultural agricultural commodities. Hence, to-be-made- consider the following
commodities. statement 1 is correct. mandatory- statements:
2. It provides the farmers NAM promotes uniformity, for-tyres-and- 1. It aims to create a pan-India
access to nationwide market, streamlining of procedures across tubes/article1 electronic trading portal for
with prices commensurate the integrated markets, removes 2079528.ece agricultural commodities.
with the quality of their information asymmetry between 2. It will be implemented as a
produce. buyers and sellers and promotes Central Sector Scheme through
Select the correct answer real time price discovery, based Agri-Tech Infrastructure Fund
using the code given below: on actual demand and supply, (ATIF).
(a) 1 only promotes transparency in auction Which of the statements given
(b) 2 only process, and access to a above is/are correct?
(c) Both 1 and 2 nationwide market for the farmer, (a) 1 only
(d) Neither 1 nor 2 with prices commensurate with (b) 2 only
quality of his produce and online (c) Both 1 and 2
payment and availability of better (d) Neither 1 nor 2
quality produce and at more
reasonable prices to the
consumer. Hence, statement 2 is
correct.

39 www.visionias.in ©Vision IAS


59 Economics With reference to the C The Union Cabinet approved the M CAA http://pib.nic.i EN IPR issues is Covered in PT365 Economy &
‘National Intellectual National Intellectual Property n/newsite/Pri always a asked in All India Test Series
Property Rights Policy’, Rights (IPR) Policy in 2016 that will ntRelease.asp important tests 2124,2218 and 2363
consider the following lay the future roadmap for x?relid=13889 issues in Consider the following
statements: intellectual property in India. This 1 Indian statements regarding India’s
1. It reiterates India’s policy shall weave in the strengths perspective. new Intellectual Property
commitment to the Doha of the Government, research and Rights policy 2016:
Development Agenda and development organizations, 1. It suggests making
the TRIPS Agreement. educational institutions, Department of Science &
2. Department of Industrial corporate entities including Technology as the nodal agency
Policy and Promotion is the MSMEs, start-ups and other for all IPR issues.
nodal agency for regulating stakeholders in the creation of an 2. It is entirely in compliant
intellectual property rights in innovation-conducive with the WTO’s agreement on
India. environment, which stimulates TRIPS.
Which of the above creativity and innovation across 3. It aims to remove
statements is/are correct? sectors, as also facilitates a stable, Compulsory Licensing clause
(a) 1 only transparent and service-oriented from India’s Patents Act.
(b) 2 only IPR administration in the country. Which of the statements given
(c) Both 1 and 2 The Policy recognizes that India above is/are correct?
(d) Neither 1 nor 2 has a well-established TRIPS- (a) 1 and 2 only
compliant legislative, (b) 2 only
administrative and judicial (c) 1 and 3 only
framework to safeguard IPRs, (d) 1, 2 and 3
which meets its international
obligations while utilizing the
flexibilities provided in the
international regime to address its
developmental concerns. It
reiterates India’s commitment to
the Doha Development Agenda
and the TRIPS agreement. Hence
statement 1 is correct.
These objectives are sought to be
achieved through detailed action
points. The action by different
Ministries/ Departments shall be
monitored by DIPP which shall be
the nodal department to
coordinate, guide and oversee
implementation and future
development of IPRs in India. Hence
statement 2 is correct.
40 www.visionias.in ©Vision IAS
60 Environment According to the Wildlife D Gharial (Gravialis gangeticus), M FA NCERT: EM The questions All India Test Series - Test 2127
(Protection) Act, 1972, which Indian Wild Ass (Equus hemionus Contemporary aims to test Which of the following species
of the following animals khur) and Wild Buffalo (Bubalus India - II , general are critically endangered?
cannot be hunted by any bubalis) are all mentioned under Chapter -2 awareness on 1. Namdapha Flying Squirrel
person except under some Schedule I for the Wildlife Forest and endangered 2. Malabar Civet
provisions provided by law? (Protection) Act, 1972. Wildlife species. Indian 3. Siberian Crane
1. Gharial Resources. wild ass and 4. Gharial
2. Indian wild ass http://envfor. the wild 5. Wild Ass
3. Wild buffalo nic.in/legis/wil buffalo are Select the correct answer using
Select the correct answer dlife/wildlife2 endangered the code given below. (a) 1 and
using the code given below: s1.pdf species. 4 only (b) 1, 2, 3 and 4 only (c)
(a) 1 only (b) 2 and 3 only 2, 3 and 5 only (d) 1, 3, 4 and 5
(c) 1 and 3 only (d) 1, 2 and 3 only
61 Polity and Which of the following D There is no legislative process D F Laxmikant - EM to test the
Governance statements is/are true of the provided to enforce the Indian Polity, basic clarity
Fundamental Duties of an Fundamental duties and they are chapter -
Indian citizen? not correlative to legal duties. Like "Fundamental
1. A legislative process has the Directive Principles, the Duties"
been provided to enforce fundamental duties are also non-
these duties. justiciable. The Constitution does
2. They are correlative to not provide for their direct
legal duties. enforcement by the courts.
Select the correct answer Moreover, there is no legal
using the code given below: sanction against their violation.
(a) 1 only (b) 2 only
(c) Both 1 and 2
(d) Neither 1 nor 2
62 Modern Consider the following pairs: B The British Indian Association was D F Ncert class EM To check the asked in All India Test Series -
History 1. Radhakanta Deb – First established on 31 October 1851. 8th-THE factual 2121 Consider the following
President of the British The first committee of the MAKING OF knowledge. pairs: Political Associations :
Indian Association association was composed of : THE Founder
2. Gazulu Lakshminarasu Raja Radhakanta Deb – NATIONAL 1.Madras Mahajan Sabha :P.
Chetty – Founder of the President, Raja Kalikrishna Deb – MOVEMENT: Ananda Charlu
Madras Mahajana Sabha Vice-President, Debendranath 1870S-1947. 2.Indian League :Surendranath
3. Surendranath Banerjee – Tagore – secretary, Digambar Banerjee
Founder of the Indian Mitra – Asst Secretary. 3.East India Association
Association In May 1884, M. :Dadabhai Naoroji Which of the
Which of the above pairs Veeraraghavachariar, G. pairs given above is/are
is/are correctly matched? Subramania Iyer and P. correctly matched?
(a) 1 only (b) 1 and 3 only Anandacharlu established the (a) 1 only (b) 1 and 3 only
(c) 2 and 3 only (d) 1, 2 and 3 Madras Mahajana Sabha. (c) 2 and 3 only (d) 1, 2 and 3
41 www.visionias.in ©Vision IAS
Gazulu Lakshminarasu Chetty was (2) Consider the following pairs:
an Indian merchant, Indian Associations Leader associated
independence activist and 1. Bombay Presidency
political activist who founded the Association :Pherozeshah
Madras Native Association. Mehta
Indian Association was the first 2. Poona Sarvajanik sabha :M.
declared Nationalist Organization G. Ranade 3. Madras
founded in British India by Mahajansabha :Subramaniya
Surendranath Banerjee and Iyer Which of the pairs given
Anand Mohan Bose in 1876. above is/are correctly
matched?
(a) 1 and 2 only (b) 3 only
(c) 1 and 3 only (d) 1, 2 and 3
63 Polity and Which one of the following B The Preamble secures to all M F Laxmikanth - EM to test the VisionIAS OPEN TEST 2 - 2226
Governance objectives is not embodied in citizens of India liberty of thought, Indian Polity, basic clarity Which among the following
the Preamble to the expression, belief, faith and Chapter - expressions is/are a part of
Constitution of India? worship, through their Preamble Preamble of the Constitution of
(a) Liberty of thought Fundamental Rights, enforceable India?
(b) Economic liberty in court of law, in case of 1. Faith
(c) Liberty of expression violation. 2. Worship
(d) Liberty of belief The ideals of liberty, equality and 3. Status
fraternity in our Preamble have 4. Freedom
been taken from the French Select the correct answer using
Revolution (1789–1799). the code given below.
(a) 1 and 3 only
(b) 1, 2 and 3 only
(c) 2 and 4 only
(d) 1, 2, 3 and 4
64 Current With reference to the C Quality Council of India (QCI) was D CAA http://www.q EN The Quality
Affairs ‘Quality Council of India set up jointly by the Government cin.org/about. Council of
(QCI)’, consider the following of India and the Indian Industry php India (QCI), on
statements: represented by the three premier its eLearning
1. QCI was set up jointly by industry associations i.e. portal
the Government of India and Associated Chambers of ‘eQuest’,
the Indian Industry. Commerce and Industry of India recently
2. Chairman of QCI is (ASSOCHAM), Confederation of launched
appointed by the Prime Indian Industry (CII) and online
Minister on the Federation of Indian Chambers of certificate
recommendations of the Commerce and Industry (FICCI), to courses in
industry to the Government. establish and operate national manufacturing
Which of the above accreditation structure and
42 www.visionias.in ©Vision IAS
statements is/are correct? promote quality through National
(a) 1 only Quality Campaign.
(b) 2 only Chairman of QCI is appointed by
(c) Both 1 and 2 the Prime Minister on
(d) Neither 1 nor 2 recommendation of the industry
to the government. The
Department of Industrial Policy &
Promotion, Ministry of Commerce
& Industry, is the nodal ministry
for QCI.
65 Current What is the purpose of A The objectives of setting up of M CAA https://rbi.org EN Small Finance PT365 Economy and All India
Affairs setting up of Small Finance small finance banks will be to .in/scripts/BS_ Banks (SFBs) test series, test 2123 (covered
Banks (SFBs) in India? further financial inclusion by- PressReleaseD have been in in explanation)
1. To supply credit to small (i) provision of savings vehicles, isplay.aspx?pri news. Which of the following
business units and d=32614 statements regarding "small
2. To supply credit to small (ii) supply of credit to small finance banks" is/are correct?
and marginal farmers business units; small and marginal 1. They are required to
3. To encourage young farmers; micro and small maintain both CRR and SLR.
entrepreneurs to set up industries; and other unorganised 2. They are not required to
business particularly in rural sector entities, through high follow Priority Sector Lending
areas. technology-low cost operations. norms.
Select the correct answer 3. Foreign investments in these
using the code given below: banks is not permitted.
(a) 1 and 2 only Select the correct answer using
(b) 2 and 3 only the code given below.
(c) 1 and 3 only (a) 1 only (b) 1 and 2 only
(d) 1, 2 and 3 (c) 2 and 3 only (d) 1, 2 and 3
66 Current With reference to ‘Asia D The 1st APMCHUD was held in D U http://mhupa. RR 6th Asia
Affairs Pacific Ministerial New Delhi, India from 13th-16th gov.in/writere Pacific
Conference on Housing and December, 2006 on the theme of addata/6thASi Conference on
Urban Development ‘A Vision for Sustainable a_Pacific_Vigy Housing and
(APMCHUD)’, consider the Urbanization in the Asia-Pacific by an_bhawan.p Urban
following statements: 2020’. ‘Emerging Urban Forms – df Development
1. The first APMCHUD was Policy Responses and Governance held in
held in India in 2006 on the Structure’ was the theme of the December
theme ‘Emerging Urban 6th Asia Pacific Conference on 2016.
Forms – Policy Responses Housing and Urban Development
and Governance Structure’. held in December 2016.
2. India hosts all the Annual 2nd APMCHUD was held in
Ministerial Conferences in Tehran, Islamic Republic of Iran,
partnership with ADB, APEC 3rd in Solo, Republic of Indonesia,
43 www.visionias.in ©Vision IAS
and ASEAN. 4th in Amman, Hashemite
Which of the statements Kingdom of Jordan and 5th in
given above is/are correct? Seoul, Republic of Korea.
(a) 1 only Further, APMCHUD is a
(b) 2 only consultative mechanism on the
(c) Both 1 and 2 promotion of sustainable
(d) Neither 1 nor 2 development of Housing and
Urban Development in the Asia-
Pacific Region established under
the aegis and support of UN-
Habitat.
67 Polity and Democracy’s superior virtue A The object of government D FA N.D. ARORA RM To test the
Governance lies in the fact that it calls according to Mill's views, being and SS philosophy or
into activity the attainment of values and Awasthy underlying
(a) the intelligence and virtues in men , their self- 'Poltical notion of a
character of ordinary men development, it is essential that Theory & basic concept
and women. the government should be of Political
(b) the methods for those whose self development is Thought'
strengthening executive being sought. This is only possible
leadership. if the people participate in the
(c) a superior individual with making and working of their
dynamism and vision. government or the government is
(d) a band on dedicated truly representative.
party workers. "Virtue and intelligence should
become genuine attributes of
character in a citizen.....and then
through participation , people at
large , would be able to control
the rulers by influencing the
character and direction of public
policy".
68 Current Which of the following is a A UPI is a payment system that M CA http://www.t EN Unified PT365 Economy
Affairs most likely consequence of allows money transfer between hehindu.com/ Payment
implementing the ‘Unified any two bank accounts by using a business/Econ Interface
Payments Interface (UPI)? smartphone. UPI allows a omy/What-is- officially
(a) Mobile wallets with not customer to pay directly from a Unified- become
be necessary for online bank account to different Payment- operational
payments. merchants, both online and Interface/artic
(b) Digital currency will offline, without the hassle of le14593189.ec
totally replace the physical typing credit card details, IFSC e
currency in about two code, or net banking/wallet
44 www.visionias.in ©Vision IAS
decades. passwords.
(c) FDI inflows will drastically
increase.
(d) Direct transfer of
subsidies to poor people will
become very effective.
69 Current The terms ‘Event Horizon’, A The terms - ‘Event Horizon’ - E F http://www.t EN All terms have
Affairs ‘Singularity’, ‘String Theory’ boundary in space-time, hehindu.com/ been in news.
and ‘Standard Model’ are ‘Singularity’ - location in space- sci-
sometimes seen in the news time where the gravitational field tech/science/
in the context of of a celestial body becomes how-a-new-
(a) Observation and infinite, ‘String Theory’ - links discovery-
understanding of the together all physical aspects, and shakes-up-
Universe ‘Standard Model’ - unifying the the-standard-
(b) Study of the solar and the four fundamental forces, are model-of-
lunar eclipses related to the observations and particle-
(c) Placing satellites in the understanding of the Universe. physics/article
orbit of the Earth 18112302.ece
(d) Origin and evolution of http://www.t
living organisms on the earth hehindu.com/
sci-
tech/science/
no-big-bang-
the-universe-
was-there-all-
along-
studies/article
6959499.ece
http://www.t
hehindu.com/
sci-
tech/science/c
v-
vishveshwara-
the-black-
hole-man-of-
india/article83
18058.ece

45 www.visionias.in ©Vision IAS


46 www.visionias.in ©Vision IAS
70 Science and With reference to agriculture D All the statements are correct. D FA https://www. RM Genome
Technology in India, how can the Genome sequencing is figuring ncbi.nlm.nih.g sequencing
technique of ‘genome out the order of DNA nucleotides, ov/pmc/articl has been in
sequencing’, often seen in or bases, in a genome—the order es/PMC48079 news due to
the news, be used in the of As, Cs, Gs, and Ts that make up 65/ various
immediate future? an organism's DNA. related
1. Genome sequencing can Genome sequencing of wild projects.
be used to identify genetic varieties of plants can be used to
markers for disease identify disease resistance and
resistance and drought drought tolerance genes in
tolerance in various crop various plants and develop new
plants. varieties of crop plants in lesser
2. This technique helps in time. Genome sequencing of cop
reducing the time required plants can be helpful in
to develop new varieties of deciphering and understanding
crop plants. 3. It can be used the host-pathogen realationship
to decipher the host- in crops.
pathogen relationships in
crops.
Select the correct answer
using the code given below:
(a) 1 only (b) 2 and 3 only
(c) 1 and 3 only (d) 1, 2 and 3
71 Polity and The main advantage of the C The Parliamentary system is also E F Laxmikanth, EM to test the VISION IAS TEST SERIES ( 35
Governance parliamentary form of known as the ‘Westminster’model Ch- Salient basic clarity Test Series Module)Test 2104,
governments is that of government, responsible Features of 2238
(a) the executive and government and cabinet the Which of the following is/are
legislature work government. The Constitution Constitution - the defining features of the
independently. establishes the parliamentary Laxmikanth parliamentary form of
(b) it provides continuity of system not only at the Centre but Government in India?
also in the states. The features of
policy and is more efficient. 1. Watertight separation of
parliamentary government in India
(c) the executive remains powers between executive,
are: (a) Presence of nominal and
responsible to the legislature and judiciary.
real executives; (b) Majority party
legislature. 2. Executive being responsible
rule, (c) Collective responsibility of
(d) the head of the the executive to the legislature, (d) to the legislature.
government cannot be Membership of the ministers in the 3. Division of powers between
changed without election. legislature, (e) Leadership of the center and state.
prime minister or the chief minister, Select the correct answer using
(f) Dissolution of the lower House the code given below.
(Lok Sabha or Assembly). (a) 1 and 2 only (b) 1 and 3
only (c) 2 only (d) 2 and 3 only

47 www.visionias.in ©Vision IAS


72 Polity and In the context of India, which A Rights are what we want others to E FA NIOS EM To test the
Governance one of the following is the do for us whereas the duties are "Democracy at basic
correct relationship between those acts which we should Work"- CH- underlying
Rights and Duties? perform for others. Thus, a right Fundamental notion behind
(a) Rights are correlative comes with an obligation to show Righst and a concept
with Duties. respect for the rights of others. Fundamental
(b) Rights are personal and The obligations that accompany Duties
hence independent of rights are in the form of duties.
society and Duties.
(c) Rights, not Duties, are
important for the
advancement of the
personality of the citizen.
(d) Duties, not Rights, are
important for the stability of
the State.
73 Polity and The mind of the makers of A In the Berubari Union case (1960), E FA Laxmikanth EM To test the
Governance the Constitution of India is the Supreme Court said that the Ch- Preamble basic
reflected in which of the Preamble shows the general of the underlying
following? purposes behind the several Constitution notion behind
(a) The Preamble provisions in the Constitution, and a concept
(b) The Fundamental Rights is thus a key to the minds of the
(c) The Directive Principles of makers of the Constitution.
State Policy
(d) The Fundamental Duties
74 Geography If you travel by road from B The shortest route from Kohima D U Atlas RR Cabinet
Kohima to Kottayam, what is (Nagaland) to Kottayam (Kerala) approved
the minimum number of passes through the following North East
States within India through states: Road Network
which you can travel, Nagaland-Assam-West Bengal- Connectivity
including the origin and the Odisha-Andhra Pradesh-Tamil Project Phase
destination? Nadu-Kerala I.
(a) 6 (b) 7
(c) 8 (d) 9
75 Polity and The Parliament of India D The Parliament exercises control E F Laxmikanth RM to test the
Governance exercises control over the over the ministers through various Ch- basic clarity
functions of the Council of devices like question hour, Parliamentary
Ministers through discussions, adjournment motion, System
1. Adjournment motion no confidence motion, etc.
2. Questions hour
3. Supplementary questions
48 www.visionias.in ©Vision IAS
Select the correct answer
using the code given below:
(a) 1 only (b) 2 and 3 only
(c) 1 and 3 only (d) 1, 2 and 3
76 Polity and With reference to the D A Private Member's bill is D FCA Laxmikanth's EM To test the
Governance Parliament of India, consider introduced by any member of Ch- basic clarity of
the following statements: Parliament other than a minister. Parliament polity
1. A private member’s bill is Its introduction in the House http://indianex knowledge
a bill presented by a Member requires one month’s notice. Its press.com/arti through the
of Parliament who is not drafting is the responsibility of the cle/opinion/col current affairs
elected but only nominated member concerned. umns/transgen
by the President of India. The last Private Member's Bill ders-persons-
2. Recently, a private passed by parliament was the bill-rajya-
member’s bill has been Supreme Court (Enlargement of sabha-private-
passed in the Parliament of Criminal Appellate Jurisdiction) member-bill-
3195872/
India for the first time in its Bill, 1968, which became an act on
http://www.th
history. August 9, 1970. The Rights of
ehindu.com/ne
Which of the statements Transgender Persons Bill, 2014, ws/national/ta
given above is/are correct? passed by the Rajya Sabha on mil-
(a) 1 only Friday is the first private nadu/jallikattu-
(b) 2 only member's bill to get the ujpper row-private-
(c) Both 1 and 2 house's approval in the past 45 member-bill-
(d) Neither 1 nor 2 years. on-bull-in-rs-
but-
withdrawn/arti
cle17517515.e
ce
77 Ancient With reference to the A Statement 1 is correct: Rigvedic D F Ancient India: EM To check basic VisionIAS All India Test Series –
History difference between the Aryans used the coat of mail and R.S. Sharma and factual 2138
culture of Rigvedic Aryans helmet in warfare whereas the page 38 THE knowledge in Which of the following metals
and Indus Valley people, people of Indus Valley Civilization WONDER Ancient have been found in Harappan
which of the following did not leave any evidence of THAT WAS History. cities?
statements is/are correct? using them. INDIA: A L 1. Bronze
1. Rigvedic Aryans used the Statement 2 is not correct: Basham page : 2. Copper
coat of mail and helmet in Various materials were used in 18 3. Iron
warfare whereas the people Harappan civilisation to make 4. Silver
of Indus Valley Civilization beads: stones like carnelian, 5. Gold
did not leave any evidence of jasper, crystal, quartz and steatite; Select the correct answer using
using them. metals like copper, bronze and the code given below.
2. Rigvedic Aryans knew gold; and shell, faience and (a) 2 and 3 only
gold, silver and copper terracotta or burnt clay. Some (b) 1, 4 and 5 only

49 www.visionias.in ©Vision IAS


whereas Indus Valley people beads were made of two or more (c) 1, 2, 3 and 5 only
knew only copper and iron. stones, cemented together, some (d) 1, 2, 4 and 5 only
3. Rigvedic Aryans had of stone with gold caps.
domesticated the horse Statement 3 is not correct:
whereas there is no evidence Evidences of the horse comes
of Indus Valley people having from a superficial level of
aware of this animal. Mohenjo daro and from a
Select the correct answer terracotta figurine from Lothal.
using the code given below: The remains of the horse have
(a) 1 only been reported from Surkotada
(b) 2 and 3 only belonging to around 2000BC. A
(c) 1 and 3 only few horse's teeth have been
(d) 1, 2 and 3 found in the lowest stratum of the
Baluchistan site of Rana Ghundal,
probably dating from several
centuries earlier than the
foundation of Harappa. This
would indicate that horse-riding
nomads found their way to N.-W.
India in small numbers long
before the Aryan invasion.
78 Current Recognition of Prior Learning A The Recognition of Prior Learning M CA http://www.t EN Recently,
Affairs Scheme’ is sometimes (RPL) is an assessment of the skills hehindu.com/ IL&FS Skills
mentioned in the news with acquired by the individual through education/car Development
reference to experience, observation and self- eers/Massive- Corporation
(a) Certifying the skills learning. The assessment, which is in-scale-and- Ltd (IL&FS
acquire by construction followed by a certification, gives innovative-in- Skills) a joint
workers through traditional an edge to an otherwise informal approach/arti initiative of
channels. worker. It provides the employee cle17287913.e IL&FS
(b) Enrolling the persons in with the confidence, social ce Education &
Universities for distance recognition and empowerment National Skills
learning programmes. that are necessary to negotiate his Development
(c) Reserving some skilled future employment. Following Corporation
jobs to rural and urban poor successful assessment, a (NSDC)
in some public sector candidate is given a financial launched
undertakings. reward, in addition to Recognition of
(d) Certifying the skills certification. Prior Learning
acquired by trainees under (RPL)
the National Skill programme in
Development Programme. the Apparel
sector under

50 www.visionias.in ©Vision IAS


Pradhan
Mantri
Kaushal Vikas
Yojana
(PMKVY) 2.0.
79 Geography From the ecological point of A Sathyamangalam forest range is a D FCA http://www.t RR The
view, which one of the significant wildlife corridor in the hehindu.com/ Sathyamangal
following assumes Nilgiri Biosphere Reserve between sci- am Tiger
importance in being a good the Western Ghats and the rest of tech/energy- Reserve was
link between the Eastern the Eastern Ghats Located in the and- in news for
Ghats and the Western Erode district of Tamil Nadu, the environment/ increase in
Ghats? Sathyamangalam Tiger Reserve is Bear- Bear
(a) Sathyamangalam Tiger the largest wildlife sanctuary in population- population.
Reserve the state. With a forest area of up-and- Also in
(b) Nallamala Forest 1,411.6 square kilometres, the counting-at- December
(c) Nagarhole National Park reserve is a part of the Nilgiri the- 2016,
(d) Seshachalam Biosphere Biosphere Reserve and lies Sathyamangal Sathyamangal
Reserve between the Western Ghats and am-Tiger- am Tiger
the rest of the Eastern Ghats. Reserve/articl Reserve eco-
Situated in a key position between e13986305.ec tourism plan
the Western and Eastern Ghats, e was revived.
the Sathyamangalam Tiger http://indiato
Reserve boasts of a wide variety day.intoday.in
of fauna. /story/sathya
The reserve also adjoins four mangalam-
other protected areas -- tiger-reserve-
Billigiriranga Swamy Temple home-to-a-
Wildlife Sanctuary, Sigur Plateau, new-breed-of-
Mudumalai National Park and bears/1/5684
Bandipur National Park. 90.html
80 Polity and One of the implications of A The term ‘equality’ means the E F Laxmikanth EM to test the
Governance equality in society is the absence of special privileges to basic clarity
absence of any section of the society, and the
(a) Privileges (b) Restraints provision of adequate
(c) Competition (d) Ideology opportunities for all individuals
without any discrimination.
81 Environment Consider the following B TRAFFIC, the wildlife trade D FCA http://www.tr EN Recent issues All India Test series - Test 2127
statements in respect of monitoring network, is the leading affic.org/traffi of wildlife Which of the following
Trade Related Analysis of non-governmental organization c-programme/ poaching and statements is/are correct with
Fauna and Flora in working globally on trade in wild organisations regard to TRAFFIC network?
Commerce (TRAFFIC): animals and plants in the context related. 1. It is a non-governmental
51 www.visionias.in ©Vision IAS
1. TRAFFIC is a bureau under of both biodiversity conservation organization working in
United Nations Environment and sustainable development. monitoring global wildlife
Programme (UNEP). TRAFFIC was established in 1976 trade.
2. The mission of TRAFFIC is by IUCN and WWF to respond to 2. It was jointly established by
to ensure that trade in wild the growing threats posed by IUCN and WWF.
plants and animals is not a illegal wildlife trade and Select the correct answer using
threat to the conservation of overexploitation. It is not a the code given below.
nature. bureau under UNEP. (a) 1 only
Which of the above (b) 2 only
statements is/are correct? (c) Both 1 and 2
(a) 1 only (b) 2 only (d) Neither 1 and 2
(c) Both 1 and 2
(d) Neither 1 nor 2
82 Polity and Which principle among the B The 42nd Amendment Act of 1976 M F Laxmikanth EM to test the VisionIAS Test Series - Test
Governance following was added to the added four new Directive Ch-Directive factual and Codes -
Directive Principles of State Principles to the original list. They Principles of basic 2104,2203,2238,2358,2364
Policy by the 42nd require the State: State Policy knowledge Which of the following
Amendment to the 1. To secure opportunities for Directive Principles was/were
Constitution? healthy development of children added to the Constitution
(a) Equal pay for equal work (Article 39). through amendments?
for both men and women 2. To promote equal justice and to 1. To secure opportunities for
(b) Participation of workers provide free legal aid to the poor healthy development of
in the management of (Article 39 A). children.
industries 3. To take steps to secure the 2. Equal pay for equal work for
(c) Right to work, education participation of workers in the men and women.
and public assistance management of industries (Article 3. To promote equal justice and
(d) Securing living wage and 43 A). to provide free legal aid to the
human conditions of work to 4. To protect and improve the poor.
workers environment and to safeguard Select the correct answer using
forests and wild life (Article 48 A). the code given below:
(a) 1 and 2 only (b) 1 and 3 only
(c) 3 only (d) 1, 2 and 3
83 Polity and Which one of the following C Rights are legal and moral M F NCERT Class EM to test the
Governance statements is correct? entitlements or claims of a person 9th Ch 6- basic clarity
(a) Rights are claims of the over other fellow beings, over 'Democratic
State against the citizens. society and over the government. Rights'
(b) Rights are privileges
which are incorporated in
the Constitution of a State.
(c) Rights are claims of the
citizens against the State.
52 www.visionias.in ©Vision IAS
(d) Rights are privileges of a
few citizens against the
many.

84 Current Which of the following gives A Global Gender Gap Index is E CA http://pib.nic.i EN India's Covered in PT365 Economy and
Affairs ‘Global Gender Gap Index’ released by World Economic n/newsite/Pri performance Social All India test sereis, test
ranking to the countries of Forum. As per the Global Gender ntRelease.asp in Global 2120, 2122
the world? Gap Report 2016, India ranks at x?relid=15510 Gender Gap Which of the following is/are
(a) World Economic Forum 87th in respect of Global Gender 3 Report 2016. the key areas of inequality
(b) UN Human Rights Council Gap Index (GGI) among 144 considered by World Economic
(c) UN Women countries of the World. As per the Forum (WEF) to compute the
(d) World Health GGR, India climbed 21 spots to Global Gender Gap Report?
Organization rank 87th in 2016, which is an 1. Health
improvement from being ranked 2.Education
at 108th in 2015. 3. Economy
4. Politics
Select the correct answer using
the code given below.
(a) 1 and 2 only
(b) 1, 2 and 3 only
(c) 2 and 3 only
(d) 1, 2, 3 and 4
85 Current Which of the following B Smart India Hackathon 2017 was a M CA http://pib.nic.i EN Smart India November 2016 VisionIAS
Affairs statements is/are correct 36 hrs non-stop digital product n/newsite/Pri Hackathon Monthly Current
regarding Smart India development competition during ntRelease.asp 2017 Affairs(Environment)
Hackathon 2017? which teams of thousands of x?relid=15982
1. It is a centrally sponsored technology students built 5
scheme for developing every innovative digital solutions for the
city of our country into problems posted by 29 different
Smart Cities in a decade. central govt. ministries/
2. It is an initiative to identify departments e.g. Ministry of
new digital technology Railways, External Affairs, Ministry
innovations for solving the of Defense, ISRO, Ministry of
many problems faced by our Tourism, Dept. of Atomic Energy,
country. etc.
3. It is a programme aimed at Smart India Hackathon 2017, has
making all the financial been launched to build Digital
transactions in our country India and to engage the youth
completely digital in a directly with nation building. For
decade. the first time, govt. departments
Select the correct answer were directly engaging with
53 www.visionias.in ©Vision IAS
using the code given below: students and challenging them to
(a) 1 and 3 only (b) 2 only build digital solutions to improve
(c) 3 only (d) 2 and 3 only their efficiency, plug revenue
leakages and corruption.
86 Economics Which of the following A The Reserve Bank of India Act, M CAA http://pib.nic.i EN Important Covered in PT365 Economy &
statements is/are correct 1934 (RBI Act) has been amended n/newsite/Pri Institutions asked in All India Test Series
regarding the ‘Monetary by the Finance Act, 2016, to ntRelease.asp which will add tests 2136 and 2017 VisionIAS
Policy Committee (MPC)? provide for a statutory and x?relid=15126 lot of value Open test 2225 Q.
1. It decides the RBI’s institutionalised framework for a 4 and With reference to Monetary
benchmark interest rates. Monetary Policy Committee it transparency Policy Committee, consider the
2. It is a 12-member body would be entrusted with the task to monetary following statements:
including the Governor of of fixing the benchmark policy policy 1. It will fix the benchmark
RBI and is reconstituted rate (repo rate) required to decisions in policy interest rates to contain
every year. contain inflation within the India. inflation within the specified
3. It functions under the specified target level. Hence target levels.
chairmanship of the Union Statement 1 is correct. 2. It will be headed by the
Finance Minister. As per the provisions of the RBI Finance minister.
Select the correct answer Act, out of the six Members of 3. The recommendations of
using the code given below: Monetary Policy Committee, MPC will be binding on the RBI.
(a) 1 only three Members will be from the Which of the statements given
(b) 1 and 2 only RBI and the other three Members above is/are correct?
(c) 3 only of MPC will be appointed by the (a) 1 only (b) 1 and 3 only
(d) 2 and 3 only Central Government. Hence (c) 2 and 3 only (d) 1, 2 and 3
statement 2 is not correct. The Consider the following
Governor of the Bank— ex officio statements regarding the
Chairperson of MPC. Hence Monetary Policy Committee
statement 3 is not correct. (MPC): It is headed by the
Governor of Reserve Bank of
India (RBI). It is entrusted with
the task of fixing the policy
interest rate to contain
inflation within the specified
target level. The Members of
the MPC appointed by the
Central Government shall hold
office for a period of four years.
Which of the statements given
above is/are correct?
(a) 2 only (b) 1 and 3 only
(c) 2 and 3 only (d) 1, 2 and 3

54 www.visionias.in ©Vision IAS


87 Art and With reference to Manipuri B Sankirtana encompasses an array M F http://ccrtindi EM Knowledge
Culture Sankirtana, consider the of arts performed to mark a.gov.in/mani about
following statements: religious occasions and various puri.php UNESCO world
1. It is a song and dance stages in the life of the Vaishnava http://pib.nic.i Heritage sites
performance. people of the Manipur plains. n/newsite/bac in India.
2. Cymbals are the only Sankirtana practices centre on the kgrounders.as
musical instruments used in temple, where performers narrate px?relid=1024
the performance. the lives and deeds of Krishna 93
3. It is performed to narrate through song and dance. Hence, https://ich.un
the life and deeds of Lord statement 1 and 3 are correct. esco.org/en/R
Krishna. In a typical performance, two L/sankirtana-
Which of the statements drummers and about ten singer- ritual-singing-
given above is/are correct? dancers perform in a hall or drumming-
(a) 1, 2 and 3 (b) 1 and 3 only domestic courtyard encircled by and-dancing-
(c) 2 and 3 only (d) 1 only seated devotees. Hence, of-manipur-
statement 2 is not correct. 00843
88 Modern Who among the following C In the British territories in the M F NCERT Class 8 EM To check the All India test series, test 2121
History was/were associated with south there was a similar move - Our Pasts - factual With regard to Warren
the introduction of Ryotwari away from the idea of Permanent Ch3 - Ruling knowledge. Hastings, consider the following
Settlement in India during Settlement. The new system that The Country statements:
the British Rule? was devised came to be known as Side PAGE 1. He facilitated the
1. Lord Cornwallis the ryotwar (or ryotwari ). It was NUMBER-29 establishment of Ryotwari
2. Alexander Read tried on a small scale by Captain System in Madras.
3. Thomas Munro Alexander Reed in some of the 2. He believed in the policy of
Select the correct answer areas that were taken over by the Orientalism as a means of
using the code given below: Company after the wars with Tipu governance of the natives.
(a) 1 only Sultan. Subsequently developed 3. The defeat of British in the
(b) 1 and 3 only by Thomas Munro, this system Second Anglo-Mysore War was
(c) 2 and 3 only was gradually extended all over a reason for his impeachment
(d) 1, 2 and 3 south India. Lord Cornwallis was in England.
asscoiated with Zamindari/ Which of the statements given
Permanent Settlement above is/are correct?
(a) 1 and 2 only (b) 1 and 3 only
(c) 2 only (d) 1, 2 and 3
89 Environment In the context of solving C Statement 1 is correct. D FA Vision IAS EM Bioremediatio All India Test series 2126, 2354
pollution problems, what Bioremediation is treatment that environment n is constantly With reference to
is/are the uses naturally occurring material. in news due to Bioremediation, consider the
advantage/advantages of organisms to break down recent following statements:
bioremediation technique? hazardous substances into less incidents of oil 1. No toxic chemicals are used
1. It is a technique for toxic or non-toxic substances. It spill and in this process.
cleaning up pollution by uses microorganisms to degrade pollution. 2. It is limited to those
55 www.visionias.in ©Vision IAS
enhancing the same organic contaminants in soil, compounds which are
biodegradation process that groundwater, sludge, and solids. biodegradable.
occurs in nature. The microorganisms break down 3. It requires anaerobic
2. Any contaminant with contaminants by using them as an conditions to be effective.
heavy metals such as energy source or cometabolizing Which of the statements given
cadmium and lead can be them with an energy source. above is/are correct?
readily and completely Statement 2 is not correct. Not all (a) 1 only
treated by bioremediation contaminants are easily treated (b) 2 and 3 only
using microorganisms. by bioremediation using (c) 1 and 2 only
3. Genetic engineering can microorganisms. For example, (d) 1, 2 and 3
be used to create heavy metals such as cadmium
microorganisms specifically and lead are not readily absorbed
designed for bioremediation. or captured by microorganisms.
Select the correct answer Statement 3 is correct. Genetic
using the code given below: engineering has been used to
(a) 1 only create organisms designed for
(b) 2 and 3 only specific purposes. For e.g.
(c) 1 and 3 only bacterium Deinococcus
(d) 1, 2 and 3 radiodurans (the most
radioresistant organism known)
has been modified to consume
and digest toluene and ionic
mercury from highly radioactive
nuclear waste.
90 Modern The Trade Disputes Act of D Trade Disputes Act (TDA), 1929 D F India's EM To check the
History 1929 provided for made compulsory the struggle for basic and
(a) the participation of appointment of Courts of Inquiry independence factual
workers in the management and Consultation Boards for , Bipin knowledge.
of industries. settling industrial disputes; chandra and
(b) arbitrary powers to the • made illegal the strikes in public Spectrum
management to quell utility services like posts, railways, page no.346
industrial disputes. water and electricity, unless each
(c) an intervention by the individual worker planning to go
British Court in the event of a on strike gave an advance notice
trade dispute. of one month to the
(d) a system of tribunals and administration;
a ban on strikes. • forbade trade union activity of
coercive or purely political nature
and even sympathetic strikes.

56 www.visionias.in ©Vision IAS


91 Polity and Local self-government can be B In January 1957, the Government E F Laxmikanth RM to check basic VisionIAS Test Series Test
Governance best explained as an exercise of India appointed a committee to Ch-Panchayati polity Series-2105,2203
in examine the working of the Raj underlying Consider the following
(a) Federalism Community Development principle statements regarding the 73rd
(b) Democratic Programme (1952) and the Constitutional Amendment act:
decentralization National Extension Service (1953) 1. It added a new part and a
(c) Administrative delegation and to suggest measures for their new schedule to the
(d) Direct democracy better working. The chairman of Constitution of India.
this committee was Balwant Rai G 2. It promoted the process of
Mehta. The committee submitted democratic decentralisation.
its report in November 1957 and 3. It has brought panchayati raj
recommended the establishment institutions under the
of the scheme of ‘democratic justiciable part of Constitution.
decentralisation’, which Which of the above statements
ultimately came to be known as are correct?
Panchayati Raj. (a) 1, 2 and 3 (b) 1 and 3 only
(c) 2 and 3 only (d) 1 and 2 only
92 Polity and Consider the following D Directive Principles of state policy M F Laxmikanth EM To check basic
Governance statements: With reference cannot act as Ch-Directive polity
to the Constitution of India, constraints/limitations on the Principles of knowledge
the Directive Principles of government because they are not State Policy
State Policy constitute enforceable and they are
limitations upon fundamental to the governance of
1. legislative function. a country. Fundamental Rights
2. executive function. operate as limitations on the
Which of the above tyranny of the executive and
statements is/are correct? arbitrary laws of the legislature.
(a) 1 only (b) 2 only
(c) Both 1 and 2
(d) Neither 1 nor 2
93 Current The term ‘Digital Single C The Digital Single Market strategy D CA http://indiane RR In May 2017,
Affairs Market Strategy’ seen in the was adopted by European Union xpress.com/ar EU published
news refers to on the 6 May 2015 and includes ticle/business/ the mid-term
(a) ASEAN 16 specific initiatives which aims economy/eu- review of the
(b) BRICS to open up digital opportunities model-well- Digital Single
(c) EU for people and business and suited-for- Market
(d) G20 enhance Europe's position as a indias- Strategy.
world leader in the digital structure-2/
economy. https://ec.eur
opa.eu/digital-
single-
57 www.visionias.in ©Vision IAS
market/en/pol
icies/shaping-
digital-single-
market
94 Geography At one of the place in India, if C Chandipur is located in Balasore D U http://indiato RR It is a
you stand on the seashore district. This beach is unique in day.intoday.in phenomenon
and watch the sea, ‘you will the whole world no where on /story/odisha- unique only to
find that the sea water earth you can find a beach where orissa- Chandipur
recedes from the shore line a the sea water retreats inside the chandipur- beach, India.
few kilometres and comes sea from 1km to 5Km every day beach-hide- Chandipur-on-
back to the shore, twice a and it again comes back to the and-seek- sea is also the
day, and you can actually shore slowly during high tide. This beach-travel- location of the
walk on the sea floor when happens twice every day. It is also india-tourism- Indian Army's
the water recedes. This known as Odisha's Hide and Seek lifetr/1/78164 Integrated
unique phenomenon is seen Beach. Bhavnagar has highest 1.html Test Range
at (a) Bhavnagar tidal range in India. http://odishat and is often
(b) Bheemunipatnam ourism.gov.in/ seen in news.
(c) Chandipur ?q=node/97
(d) Nagapattinam
95 Current With reference to the B Salient features of Prohibition of D CA http://www.b EN Benami PT365 Economy
Affairs ‘Prohibition of Benami Benami Property Transaction Act, usiness- Property
Property Transaction Act, 1988 (PBPT Act)- standard.com/ Transactions
1988 (PBPT Act)’, consider - The PBPT Act defines benami article/econo Act, 1988 was
the following statements: transactions, prohibits them and my-policy/ls- amended and
1. A property transaction is further provides that violation of okays- renamed as
not treated as a benami the PBPT Act is punishable with changes-to- renamed as
transaction if the owner of imprisonment and fine. The PBPT benami- Prohibition of
the property is not aware of Act prohibits recovery of the property-law- Benami
the transaction. property held benami from 11607270153 Property
2. Properties held benami benamidar by the real owner. 3_1.html Transactions
are liable for confiscation by - The definition of a benami http://pib.nic.i Act, 1988
the Government. transaction has been widened to n/newsite/Pri (PBPT Act).
3. The Act provides for three include a transaction made in a ntRelease.asp
authorities for investigations fictitious name; where the owner x?relid=15308
but does not provide for any is not aware or denies knowledge 5
appellate mechanism. of the ownership of the property
Which of the statements or the person providing the
given above is/are correct? consideration for the property is
(a) 1 only (b) 2 only not traceable.
(c) 1 and 3 only - Properties held benami are liable
(d) 2 and 3 only for confiscation by the
58 www.visionias.in ©Vision IAS
Government without payment of
compensation.
- An appellate mechanism has
been provided under the PBPT Act
in the form of Adjudicating
Authority and Appellate Tribunal.
96 Environment Due to some reasons, if C Statement 1 and 3 are correct. D FA http://www.t RR To check the
there is a huge fall in the Butterflies are pollinating insects. hehindu.com/ basic concepts
population of species of They help in pollination of many news/cities/Ti and its
butterflies, what could be its flowering plants. ruchirapalli/w applicability.
likely Butterflies also act as a lower hen-
consequence/consequences? member of the food chain. A butterflies-
1. Pollination of some plants number of animals, including birds take-a-
could be adversely affected. and mice feed on butterfly. As hit/article792
2. There could be a drastic populations of butterfly diminish, 8831.ece
increase in the fungal so will populations of birds and
infections of some cultivated other animals that rely on them as
plants. a food source. This loss of the
3. It could lead to a fall in the butterfly is the beginning of the
population of some species “butterfly effect.”
of wasps, spiders and birds. Statement 2 is not correct.
Select the correct using the
code given below:
(a) 1 only (b) 2 and 3 only
(c) 1 and 3 only
(d) 1, 2 and 3
97 Environment It is possible to produce B Statement 1 is incorrect. D FCA http://www.fa RR In news
algae based biofuels, but Production of algal biofuel is o.org/3/a-
what is/are the likely possible both in seas and on ak333e.pdf
limitation(s) of developing continents. They can grow on http://www.t
countries in promoting this marginal or non-crop land and hehindubusin
industry? also on brackish or polluted essline.com/e
1. Production of algae based water. Land based systems are conomy/agri-
biofuels is possible in seas more developed then sea based business/india
only and not on continents. systems. -can-be-a-
2. Setting up and engineering Statements 2 and 3 are correct. world-leader-
the algae based biofuels Developing and engineering ABB in-algal-
production requires high technology requires a high level of farming-says-
level of expertise/ expertise until construction is us-
technology until the finished. expert/article
construction is complete. Innovation for higher productivity 9449569.ece
59 www.visionias.in ©Vision IAS
3. Economically viable also requires some knowledge
production necessitates the and/or experience. All Algal based
setting up of large scale biofuel concepts require
facilities which may raise significant capital investment.
ecological and social Access to this technology by the
concerns. poor may be difficult. Large-scale
Select the correct answer facilities are more economically
using the code given below: viable, but are also more likely to
(a) 1 and 2 only have higher social and ecological
(b) 2 and 3 only impacts.
(c) 3 only (d) 1, 2 and 3
98 Current Which of the following are A There are two components of the D CA http://pib.nic.i EM The Union
Affairs the objectives of ‘National National Nutrition Mission as n/newsite/Pri Budget 2016
Nutrition Mission’? follows: ntRelease.asp increased
1. To create awareness 1. Information, Education and x?relid=10319 allocations for
relating to malnutrition Communication (IEC) Campaign 2 the National
among pregnant women and against malnutrition http://timesof Nutrition
lactating mothers. 2. Multi-sectoral Nutrition india.indiatim Mission and
2. To reduce the incidence of Programme es.com/india/ establishment
anaemia among young The key objectives of these Nutrition- of new labs
children, adolescent girls and programmes are as under: Mission-gets- for testing
women. - To create awareness relating to new- food given
3. To promote the malnutrition amongst pregnant boost/articles under the
consumption of millets, women, lactating mothers, how/5124807 scheme.
coarse cereals and promote healthy lactating 8.cms Global
unpolished rice. practices and importance of Nutrition
4. To promote the balanced nutrition; Report
consumption of poultry eggs. - To improve maternal and child
Select the correct answer under-nutrition in 200 high
using the code given below: burdened districts and to prevent
(a) 1 and 2 only and reduce the under-nutrition
(b) 1, 2 and 3 only prevalent among children below 3
(c) 1, 2 and 4 only years;
(d) 3 and 4 only - To reduce incidence of anaemia
among young children, adolescent
girls and women.
99 Modern Consider the following B To improve the lot of the factory D F India's EM To check the
History statements: workers in towns, he passed the struggle for basic
1. The Factories Act, 1881 first Factory Act in 1881. The Act independence knowledge.
was passed with a view to fix prohibited the employment of , Bipin
the wages of industrial children under the age of seven, chandra,
60 www.visionias.in ©Vision IAS
workers and to allow the limited the number of working Chapter - the
workers to form trade hours for children below the age Indian
unions. of twelve and required that working class
2. N. M. Lokhande was a dangerous machinery should be and the
pioneer in organizing the fenced properly. national
labour movement in British The Act also made provision for movment
India. one hour rest during the working
Which of the above period and four days leave in a
statements is/are correct? month for the workers. Inspectors
(a) 1 only were appointed to supervise the
(b) 2 only implementation of these
(c) Both 1 and 2 measures. Hence there is no
(d) Neither 1 nor 2 provision for fixed wage and
formation of trade unions.
Narayan Meghaji Lokhande was a
pioneer of the labour movement
in India. He is remembered not
only for ameliorating the working
conditions of textile mill-hands in
the 19th century but also for his
courageous initiatives on caste
and communal issues.
100 Environment In the context of mitigating D Carbon sequestration is the M FCA http://www.t RR Carbon All India Test Series Tests 2138,
the impending global process involved in carbon hehindu.com/ sequestration 2132
warming due to capture and the long-term storage sci- and its Oceans are considered to be a
anthropogenic emissions of of atmospheric carbon dioxide. tech/energy- methods are big reservoir of carbon sink. In
carbon dioxide, which of the Geological sequestration involves and- often seen in this context, which of the
following can be potential the storage of CO2 underground environment/ news. UPSC following may provide for the
sites for carbon in depleted oil and gas reservoirs, Carving-out-a- has gone one mechanism of ocean
sequestration? saline formations or deep, un- model-for- step further to sequestration?
1. Abandoned and minable coal beds. enhancing- ask potential 1. Introduction of iron into
uneconomic coal seams CO2- sites for oceans
2. Depleted oil and gas sinks/article14 geological 2. Introduction of urea into
reservoirs 001561.ece carbon oceans
3. Subterranean deep saline sequestration. 3. Direct injection of carbon
formations dioxide in the ocean
Select the correct answer Select the correct answer using
using the code given below: the code given below.
(a) 1 and 2 only (b) 3 only (a) 1 and 2 only (b) 2 and 3 only
(c) 1 and 3 only (d) 1, 2 and 3 (c) 1 and 3 only (d) 1, 2 and 3

61 www.visionias.in ©Vision IAS


TOPIC

Topic Number of Questions


Modern History 7
Medieval History 1
Ancient History 1
Art and Culture 5
Science and
Technology 4
Economics 6
Geography 8
Polity and
Governance 22
Environment 13
Current Affairs 33
Grand Total 100

62 www.visionias.in ©Vision IAS


DIFFICULTY

Difficulty Number of Questions

E - Easy 27

M - Medium 37

D - Difficult 36

63 www.visionias.in ©Vision IAS


NATURE

Nature Number of Questions

CA – Current Affair 15

CAA – Current Affair

Applied 20

F – Fundamental 31

FA – Fundamental

Applied 15

FCA – Fundamental and

Current Affair 10

U - Unconventional 9

64 www.visionias.in ©Vision IAS


SOURCE TYPE

Source Type Number of Questions

EM – Essential Material 37

RM – Reference Material 7

EN – Essential News 37

RR – Random Read 19

Grand Total 100

65 www.visionias.in ©Vision IAS


SECTIONWISE DIFFICULTY

66 www.visionias.in ©Vision IAS


Easy Medium Difficult Total

Economics 2 4 0 6

Ancient History 0 0 1 1

Modern History 1 3 3 7

Medieval History 0 0 1 1

Science and Technology 0 1 3 4

Geography 0 3 5 8

Polity and Governance 10 8 4 22

Environment 3 4 6 13

Art and Culture 1 1 3 5

Current Affairs 10 13 10 33

Total 27 37 36 100

67 www.visionias.in ©Vision IAS

You might also like